Professional Conduct MCQS Flashcards

1
Q

Identify whether the following scenario indicates a high or low risk of money laundering:

Your client, a British national, is purchasing a flat for his daughter. He deposits £500,000 in your firm’s client account for the purchase from a bank account in Bermuda. The next day he rings you to inform you that the purchase is not going ahead. He asks you to transfer the £500,000 to his wife’s bank account in the Cayman Islands

A) Low risk. Neither Bermuda nor the Cayman Islands are on the FATF list of countries with a high risk of money laundering.

B) High risk, because the client’s deposit came from an offshore bank account in a tax haven, the transaction has been cancelled at short notice and the client is asking you to transfer the money to a different account, also offshore in a different tax haven

C) High risk because Bermuda and the Cayman Islands are on the European Commission’s list of high risk jurisdictions.

A

B) High risk, because the client’s deposit came from an offshore bank account in a tax haven, the transaction has been cancelled at short notice and the client is asking you to transfer the money to a different account, also offshore in a different tax haven

(Correct. There are a number of risk factors here which should alert you to the high risk of money laundering. The client has deposited a large sum of money in cash in your firm’s account. The money came from an offshore bank account in a tax haven. The transaction has been cancelled at short notice and the client is asking you to transfer the money to a different account, also offshore in a different tax haven.)

How well did you know this?
1
Not at all
2
3
4
5
Perfectly
2
Q

Identify whether the following scenario indicates a high or low risk of money laundering:

Your client, a French national, living in the UK is setting up a trust for her grandchildren. She plans to invest £50,000 in the trust over the course of two years.

A) High risk because the client is investing a large sum of money in a trust which is a suspicious area of activity because it could be a vehicle for money laundering

B) High risk because although the client is a French national, she is setting up a trust in the UK

C) Low risk because France is not a high risk country and there appears to be a legitimate reason for the client to set up the trust

A

C) Low risk because France is not a high risk country and there appears to be a legitimate reason for the client to set up the trust

(Correct. A client setting up a trust, does have the potential to be a vehicle for money laundering and you will need to carry out customer due diligence on the client, as you will see later in this topic. However there is nothing to suggest a high risk of money laundering here, France is not a high risk country and there appears to be a legitimate reason for the client to set up the trust)

How well did you know this?
1
Not at all
2
3
4
5
Perfectly
3
Q

Identify whether the following scenario indicates a high or low risk of money laundering:

Your client, a South African national, has instructed you on a litigation claim and asked you to commence proceedings against her neighbour. The neighbour contacts you and wants to pay £100,000 into your firm’s client account to settle the matter.

A) High risk because the dispute being settled early is an indicator that the litigation might be a sham. Also the other party is seeking to pay a large sum of money into your firm’s bank account

B) High risk because South Africa is on the European Commission’s list of high risk jurisdictions

C) Low risk because litigation is not a high risk activity from a money laundering perspective

A

A) High risk because the dispute being settled early is an indicator that the litigation might be a sham. Also the other party is seeking to pay a large sum of money into your firm’s bank account

(Correct. There are a number of risk factors here which should alert you to the high risk of money laundering. A dispute being settled early and easily should alert you to the risk of this being a sham litigation, and the other party seeking to pay a large sum of money into your firm’s account is another warning sign. South Africa is not on the list of high risk countries, but there are enough other factors here to indicate a high risk of money laundering)

How well did you know this?
1
Not at all
2
3
4
5
Perfectly
4
Q

Which ONE of the following activities does NOT fall within the definition of ‘regulated sector’ in Schedule 9 of PoCA?

A) Advising a client on the purchase of a property

B) Advising a client on setting up a trust for their grandchildren

C) Advising a client on an employment dispute

A

C) Advising a client on an employment dispute

(Correct. ‘Regulated sector’ as defined in Schedule 9 PoCA includes “participating in financial and real property transactions concerning:
the buying and selling of real property or business entities
the managing of client money, securities or other assets,
the opening or management of bank, savings or securities accounts;
the organisation of contributions necessary for the creation, operation and management of companies;
the creation, operation or management of trusts, companies or similar structures
Advising a client on the purchase of a property and on setting up a trust would fall within participating in financial and real property transactions. However advising on litigation matters (including an employment dispute) does not fall within the definition of regulated sector, as set out in Schedule 9)

How well did you know this?
1
Not at all
2
3
4
5
Perfectly
5
Q

You are an apprentice in the property department of a law firm and you are contacted by a new client who has never instructed the firm before. The client contacts you directly asking for details of the firm’s client account. He wants to send the sum of £6,000,000 from an account in Bermuda to the firm’s client account in readiness for a quick completion of his purchase of a property in London, on which he is just about to instruct the firm (the ‘Purchase’). He insists that he wants you to act personally on the purchase.

You explain to the Client that you are not able to give the Client the firm’s client account details. The Client tells you he will call in at the office tomorrow and he wants you to give him the firm’s client account details then.

Which ONE of the following statements is CORRECT?

A) When the Client arrives at the office, you should not give him the client account details, but tell him that you have reported the matter to the MLRO (money laundering reporting officer) because you suspect money laundering

B) You should meet the Client and give him the account details. There are no grounds for suspicion here as it is not unusual for a property in London to cost £6,000,000

C) You should meet the Client and give him the client account details – so as not to arouse suspicion

D) You should immediately make a disclosure to the firm’s MLRO (money laundering reporting officer) under the Proceeds of Crime Act 2002 (‘PoCA’) that you suspect the money is criminal property. You must wait to receive appropriate consent from the MLRO before carrying out the Client’s instructions

A

D) You should immediately make a disclosure to the firm’s MLRO (money laundering reporting officer) under the Proceeds of Crime Act 2002 (‘PoCA’) that you suspect the money is criminal property. You must wait to receive appropriate consent from the MLRO before carrying out the Client’s instructions

(Correct. There are a number of risk factors here which should alert you to the risk of the Client attempting to launder money, notably: the client is new to your firm and has sent a large amount of money (£6 million) in cash to your firm’s client account, the client wants the transaction to complete quickly and he wants you (an inexperienced apprentice) to act personally on the transaction.
If you carry out the Client’s instruction you are at risk of committing an offence under s. 327 (transferring criminal property), s. 328 (becoming concerned in an arrangement which facilitates the acquisition…use or control of criminal property) and/or s. 329 PoCA (acquires, uses or has possession of criminal property).
You will also commit an offence under s. 330 PoCA if you fail to disclose your suspicion to the firm’s nominated officer. You should not take further action until you receive appropriate consent from the MLRO, s. 327(2)(a), s. 328(2)(a) PoCA.
You should be careful that you do not commit the offence of tipping off the client under s. 333A PoCA)

How well did you know this?
1
Not at all
2
3
4
5
Perfectly
6
Q

Xavier Gaston is a French national. He has never instructed Price Prior before, where you are currently an apprentice in the corporate department. Xavier has sent the sum of £2,000,000 from an account in Jersey to Price Prior’s client account in readiness for a quick completion of his acquisition of all the shares in FWL (Retail) Ltd, which he is just about to instruct you on (the ‘Acquisition’). Xavier insists that he wants you to act personally on the deal.

If you proceed with Xavier’s instructions and complete the Acquisition WITHOUT making an authorised disclosure to Price Prior’s MLRO, which ONE of the following statements is CORRECT in relation to offences that you have committed under the Proceeds of Crime Act 2002 (‘PoCA’)?

A) You are likely to have committed an offence under sections 329 and 330 PoCA

B) You are not likely to have committed an offence under PoCA

C) You are likely to have committed a direct involvement offence under section 329 PoCA, but not a non-direct involvement offence because you are not working in the regulated sector

D) You are likely to have committed an offence under section 333A PoCA.

A

A) You are likely to have committed an offence under sections 329 and 330 PoCA

(Correct. There are a number of risk factors here which should alert you to the risk of Xavier attempting to launder money, notably: the client is new to your firm and has sent a large amount of money (£2 million) in cash to your firm’s client account, the client wants the transaction to complete quickly and he wants you (an inexperienced apprentice) to act personally on the transaction.
If it is later discovered that the Acquisition was part of a money laundering operation, you will have had possession of criminal property (the proceeds of crime) so you are likely to have committed an offence under s. 329 PoCA 2002.
In addition, you will have committed an offence under s. 330 PoCA 2002 because there is sufficient grounds for you to suspect that Xavier is engaged in money laundering, for the reasons set out above. Under s. 330 PoCA 2002 it is an offence for someone practising in the regulated sector (which you are, as you are involved in buying a business entity) not to make a disclosure to your firm’s MLRO. You are practising in the regulated sector because you are buying a business entity which falls within the definition of “participating in financial and real property transactions concerning: the buying and selling of real property or business entities”.
You will not have committed an offence under s. 333A PoCA 2002 because you have not done anything to tip Xavier off to the fact that you suspect he may be attempting to launder money)

How well did you know this?
1
Not at all
2
3
4
5
Perfectly
7
Q

Identify whether the following scenario requires standard CDD, enhanced CDD or ongoing monitoring:

You have been instructed by a company Autolext Corporation (‘Autolext’) on the purchase of 60% of the shares in a data analytics company, Data Inc for £2 million. Data Inc is registered in the Cayman Islands. Autolext is registered in Bermuda and has a majority shareholder who is Iranian. Autolext has asked to you set up a wholly owned subsidiary, Autolext Data Limited which will be the vehicle that purchases the shares in the Data Inc. Autotext will pay cash for the shares.

A) Standard CDD

B) Enhanced CDD

C) Ongoing monitoring

A

B) Enhanced CDD

(Correct. There are a number of factors that suggest that there is a high risk of money laundering in this scenario. The client is registered in Bermuda, an off-shore tax haven, and its majority shareholder is from a high risk country, Iran. The value of the transaction is high and is being funded in cash. The client is also asking you to establish a subsidiary for the sole purpose of purchasing the shares, which could indicate a transaction that is unusually complex with no apparent purpose. The target is also registered in an off-shore tax haven, the Cayman Islands. You would need to make further enquiries to satisfy yourself as to the source of the £2 million, the identity of the majority shareholder and that the transaction is genuine)

How well did you know this?
1
Not at all
2
3
4
5
Perfectly
8
Q

Identify whether the following scenario requires standard CDD, enhanced CDD or ongoing monitoring:

Plaza Properties Limited is a private limited company and an existing client of your firm and regularly instructs you on the acquisition and sale of properties in its portfolio. You have received a new instruction to act for it on the acquisition of the freehold of an office building near Maidenhead for £10 million. Part of the acquisition is funded by a loan.

A) Standard CDD

B) Ongoing monitoring

C) Enhanced CDD

A

B) Ongoing monitoring

(Correct. This is an existing client whom the firm acts for on a regular basis. The firm will already have carried out CDD on the client. The firm will need to carry out ongoing monitoring, by checking that the transaction is consistent with the firm’s knowledge of the client and checking at regular intervals that the identification documents it holds for the client are up to date. There is nothing on these facts to suggest that this client present a high risk of money laundering)

How well did you know this?
1
Not at all
2
3
4
5
Perfectly
9
Q

Identify whether the following scenario requires standard CDD, enhanced CDD or ongoing monitoring:

Your client, a British national, is purchasing a flat for her daughter for a price of £750,000. She is funding 50% of the purchase in cash from the sale of some shares, and 50% by a mortgage with a high street bank. You have not acted for the client before and you expect that this will be a one-off transaction for the client.

A) Ongoing monitoring

B) Standard CDD

C) Enhanced CDD

A

B) Standard CDD

(Correct. There is nothing on these facts to suggest that this client present a high risk of money laundering. The is not from a high risk country and is funding 50% of the purchase through a mortgage. As this is a one-off transaction, it will be an occasional transaction and you should obtain standard CDD on the client before you start working for her)

How well did you know this?
1
Not at all
2
3
4
5
Perfectly
10
Q

You are a solicitor in the corporate department of Price Prior. Your client wishes to buy 50% of the shares in a company as an investment. Your client has instructed you to advise her on the acquisition (including on the merits).

Which one of the following statements is correct regarding authorisation under FSMA?

A) There is a specific exclusion relating to advising on the merits that will allow you to act

B) FSMA does apply and you will need to be authorised by the FCA to act for the client on the acquisition

C) You will not need to be authorised by the FCA to act for the client because acting for a client who is acquiring shares in a company is not a specified activity under Article 53(1) RAO

D) Shares are not a specified investment, therefore FSMA does not apply

E) You will not need to be authorised by the FCA to act for the client because your advice will be excluded from FSMA under Article 70 RAO

A

E) You will not need to be authorised by the FCA to act for the client because your advice will be excluded from FSMA under Article 70 RAO

How well did you know this?
1
Not at all
2
3
4
5
Perfectly
11
Q

Which one of the following would not be classified as financial services and therefore not be regulated under FSMA?

A) Arranging investment products on behalf of clients

B) Dealing in investment products on behalf of clients

C) Advising on the acquisition of a business

D) Advising on investment products on behalf of clients

A

C) Advising on the acquisition of a business

How well did you know this?
1
Not at all
2
3
4
5
Perfectly
12
Q

You recently acted for a client in connection with the sale of his personal training business and that sale has now concluded.

The client tells you that following the sale, he now has £50,000 cash and he would like some advice about how to invest this money, specifically whether he should buy shares in XYZ Plc. He wants to know whether shares are generally a good investment at the moment and, in particular, whether or not he should buy the shares in XYZ.

Which one of the following statements is correct in relation to the above scenario?

A) The advice is not likely to be incidental under s 327 FSMA

B) The advice is likely to be complementary under Scope Rule 2

C) You would be able to give the client generic advice on the rights attaching to shares because this advice does not fall within Article 53(1) RAO

D) It is likely that Article 67 of the Financial Services and Markets Act (Regulated Activities) Order 2001 (‘RAO’) would apply to your advice

E) You would be free to advise about the merits of the shares in XYZ Ltd because you could rely on the exclusion under Art 70 RAO

A

C) You would be able to give the client generic advice on the rights attaching to shares because this advice does not fall within Article 53(1) RAO

How well did you know this?
1
Not at all
2
3
4
5
Perfectly
13
Q

Which of the following is not a key part of the criminal justice process as discussed in this element?

A) Appeal

B) Trial

C) Plea

D) Arrest

E) The prevention of crime

A

E) The prevention of crime

(Correct. The key elements of the criminal justice process discussed in this element were: arrest/ requisition, plea, trial, sentencing and appeals. The prevention of crime is more of a matter for criminology studies but does become a factor when sentencing someone found guilty of a crime)

How well did you know this?
1
Not at all
2
3
4
5
Perfectly
14
Q

All of the following bodies can bring prosecutions. However, which of the following is the main body which brings prosecutions in England and Wales?

A) Transport for London

B) Health and safety executive

C) Crown Prosecution Service

D) Railway operators

E) Private individuals

A

C) Crown Prosecution Service

(Correct. Other bodies such as Transport for London, railway operators and the Health and Safety Executive prosecute their own matters. Private individuals can bring prosecutions too but the CPS has the right to intervene and take over such cases)

How well did you know this?
1
Not at all
2
3
4
5
Perfectly
15
Q

What is the main statute that governs police conduct in criminal investigations?

A) Police Powers Act 1984

B) Governance of Police Powers Act 1984

C) Police and Criminal Evidence Act 1984

A

C) Police and Criminal Evidence Act 1984

(Correct. Commonly known as ‘PACE’ which is accompanied by Codes of Practice)

How well did you know this?
1
Not at all
2
3
4
5
Perfectly
16
Q

What does trial on indictment mean?

A) Trial by jury in the Crown Court

B) Trial by magistrates in a magistrates’ court

C) Trial by jury in a magistrates’ court

A

A) Trial by jury in the Crown Court

(Correct. Either-way offences and indictable only offences can be heard before a jury)

How well did you know this?
1
Not at all
2
3
4
5
Perfectly
17
Q

What are the three classifications of offences?

A) Property offences, fatal offences and non-fatal offences against the person

B) Low, medium and high

C) Summary, either-way and indictable only

A

C) Summary, either-way and indictable only

(Correct. Either-way offences are also known as ‘indictable’ as they are capable of being tried on indictment)

How well did you know this?
1
Not at all
2
3
4
5
Perfectly
18
Q

If the statute provides a maximum penalty on summary conviction but does not provide for a penalty on conviction on indictment, what type of offence is it?

A) Summary offence

B) Indictable offence

C) Either-way offence

A

A) Summary offence

(Correct. Reading the statute is one of the ways to determine the classification of an offence)

How well did you know this?
1
Not at all
2
3
4
5
Perfectly
19
Q

If the statute provides one penalty for summary conviction and a different one for conviction on indictment, what type of offence is it?

A) Indictable only

B) Summary

C) Either-way

A

C) Either-way

(Correct. A close reading of the statute is one way to tell whether an offence is either-way. An offence can also be either-way if listed in Schedule 1, Magistrates’ Courts Act 1980)

How well did you know this?
1
Not at all
2
3
4
5
Perfectly
20
Q

If the statute provides only for a penalty on conviction on indictment or is a common-law offence what type of offence is it?

A) Indictable-only

B) Summary

C) Either-way

A

A) Indictable-only

(Correct. Reading the statute and knowledge of the common-law offences helps determine the classification of offences)

How well did you know this?
1
Not at all
2
3
4
5
Perfectly
21
Q

What is the function of a jury in a Crown Court trial?

A) Tribunal of law

B) Tribunal of fact

C) Sums up the case

A

B) Tribunal of fact

(Correct. There are certain circumstances in which the judge may also act as a tribunal of fact such as when hearing a submission of no case to answer, a voir dire or a Newton hearing which will be considered in separate elements)

How well did you know this?
1
Not at all
2
3
4
5
Perfectly
22
Q

How are the roles of tribunal of fact and tribunal of law dealt with in the magistrates’ court?

A) The magistrates are the tribunal of fact and tribunal of law

B) The magistrates are the tribunal of fact and the legal advisor is the tribunal of law

C) The magistrates are the tribunal of law, there is no tribunal of fact

A

A) The magistrates are the tribunal of fact and tribunal of law

(Correct. This is arguably an unsatisfactory position when the magistrates make a ruling of law that certain evidence is inadmissible but hear the evidence in order to make that decision. In contrast, in the Crown court it would be the judge that would make a ruling of law that certain evidence is inadmissible and the jury would never hear the evidence)

How well did you know this?
1
Not at all
2
3
4
5
Perfectly
23
Q

How many Crown Courts are there in England and Wales?

A) 10-19

B) One

C) Less than 10

D) More than 20

A

B) One

(Correct. The Crown Court is regarded as a single court that sits in different places)

How well did you know this?
1
Not at all
2
3
4
5
Perfectly
24
Q

Which judges can sit in a summary trial?

A) Lay justices, District Judges or Deputy District Judges

B) Lay justices or District Judges

C) District Judges

D) Lay justices

A

A) Lay justices, District Judges or Deputy District Judges

(Correct. Lay justices (also known as Justices of the Peace), District Judges (a salaried judge) or a Deputy District Judge (a solicitor or barrister with a part-time judicial post) can conduct trials in the magistrates’ court)

How well did you know this?
1
Not at all
2
3
4
5
Perfectly
25
Q

You represent a man suspected of the offence of battery and is charged with common assault. He is currently out of work and so receives welfare benefits.

Which of the following best explains whether you client is eligible for public funding?

A) He will automatically pass the means test because he is in receipt of welfare benefits but he must also pass the merits test to be eligible

B) He will receive public funding because the offence of battery is an imprisonable offence

C) He will pass the means test if he can show that his income and capital are below a certain limit

D) He will automatically qualify for public funding because he is in receipt of welfare benefits

E) He will have to pass both the means test and the merits test in order to be granted public funding

A

A) He will automatically pass the means test because he is in receipt of welfare benefits but he must also pass the merits test to be eligible

(He is passported through the means test because of his receipt of welfare benefits, but he will still have to show that it is in the interests of justice for him to receive public funding for his defence.
The other answers are plausible but incorrect:
- He does not need to pass the means test – he automatically passes it
- He won’t automatically qualify for public funding because he still has to pass the merits test
- Whether the offence is imprisonable or not is not a deciding factor)

How well did you know this?
1
Not at all
2
3
4
5
Perfectly
26
Q

Your client is a university professor who lives alone and earns £90,000. He is suspected of the offence of battery on a student and is charged with common assault. He is due to appear before the magistrates’ court He has no previous offences and intends to plead not guilty.

Is your client likely to obtain a representation order that grants public funding for his defence?

A) He will obtain a representation order because it is likely he will lose his livelihood

B) He will not obtain a representation order because it is not in the interests of justice that he be represented

C) He will not obtain a representation order as will fail the means test

D) He will not obtain a representation order because he is not in receipt of benefits

E) He will obtain a representation order because it is likely that he will suffer serious damage to his reputation

A

C) He will not obtain a representation order as will fail the means test

(In light of the university professor’s salary and the size of his household (he lives alone) he will be ineligible for public funding because his weighted gross annual income will be above the upper threshold i.e. £22, 325.
The other answers while plausible are incorrect:
· While he will suffer serious damage to his reputation, might lose his livelihood and therefore it might be in the interests of justice that he be represented, he will not pass the means test.
· While he is not in receipt of benefits that does not mean he will automatically fail the means test)

How well did you know this?
1
Not at all
2
3
4
5
Perfectly
27
Q

Your client has been charged with burglary. She is in receipt of benefits and therefore is passported through the means test. You are assisting your client in completing the CRM 14 form.

She informs you that she has numerous convictions for burglary, including one last year when she was given a suspended sentence. She denies the offence and says she has an alibi. However, your client is worried about what the arresting officer will say. The arresting officer said she had admitted the offence on arrest, when she did not say anything. She’s also worried about her daughter who is only 12 and will be left on her own if she is sent to prison.

Which of these is not a matter to include in the CRM 14 form?

A) She is likely to lose her liberty

B) It is in the interests of her daughter that she is represented

C) She has been given a sentence that is suspended which will be activated if she is convicted of the current offence

D) An alibi witness will need to be traced and interviewed

E) The proceedings will involve the cross examination of a police officer

A

B) It is in the interests of her daughter that she is represented

(For the purposes of the merits/ interests of justice test, the interests of another person does not include family members impacted by a sentence.
The other answers while plausible are not correct. All could be included in the relevant sections of the CRM 14 form)

How well did you know this?
1
Not at all
2
3
4
5
Perfectly
28
Q

In which of the following situations can you accept instructions from the person identified?

A) A majority shareholder instructing you to sell the company’s headquarters.

B) From a man on behalf of his elderly mother in selling her home to pay for care fees. His mother has lost mental capacity and so is unable to instruct you.

C) A branch manager of a shop on behalf of a nationwide retail company in buying premises neighbouring the branch.

D) From a woman on behalf of her brother in the sale of his flat as he is travelling overseas.

E) From a woman on behalf of her and her civil partner. You met them both initially in which they confirmed that you can accept instructions from either of them on behalf of the other.

A

E) From a woman on behalf of her and her civil partner. You met them both initially in which they confirmed that you can accept instructions from either of them on behalf of the other.

(Correct. As they have given you authority, you can rely on that)

How well did you know this?
1
Not at all
2
3
4
5
Perfectly
29
Q

You are acting for a couple purchasing a house. You are also acting for their lender, a high street bank. The couple do not want you to tell the lender that they intend to let out the property as they know the lender would then insist that the mortgage interest rate would increase from the mortgage rate they have been offered on the basis of the house being lived in by the couple as their main residence. What is the professional conduct issue here?

A) You have a duty to report the couple’s plans to the lender.

B) You advise the clients to proceed as it is unlikely that the lender will ever discover that the property is let.

C) Your duty of confidentiality to the couple overrides your duty to disclose. You must inform the lender that you cannot now act for professional reasons.

D) You have a duty to immediately report to the relevant authority that the clients are committing mortgage fraud.

A

C) Your duty of confidentiality to the couple overrides your duty to disclose. You must inform the lender that you cannot now act for professional reasons.

How well did you know this?
1
Not at all
2
3
4
5
Perfectly
30
Q

You are acting for a landlord that is granting a lease to a company. You have drafted the lease and sent a copy of the lease directly to the company as it had not yet instructed solicitors. One of the directors of the company calls you and asks you to act for the company as well as the landlord to save costs. The director says that the rent is agreed.

**Which one of the following statements is correct?*

A) The landlord and the company have a substantially common interest as all the key terms in the lease have been agreed so you, personally would be able to act for both of them by relying on CCS 6.2(a) so long as you could show you had satisfied all the elements of CCS 6.2 (i) to (iii).

B) If you, personally, were to act for both the landlord and the company, there is a risk you would breach CCS 6.2

C) As this transaction falls within CCS 6.1 you personally would not be able to act for both the landlord and the tenant

D) GCL and NWL are competing for the same objective as they both have the same objective of wanting the lease to be granted so you, personally, could act for them both by relying on CCS 6.2(b) so long as you could show you had satisfied all the elements of CCS 6.2 (i) to (iii).

A

B) If you, personally, were to act for both the landlord and the company, there is a risk you would breach CCS 6.2

(Correct. In negotiating the lease you will not be able to act in both the landlord’s and the company’s best interests leading to a conflict of interests under CCS 6.2. There are more clauses in the lease to be agreed than just the rent)

How well did you know this?
1
Not at all
2
3
4
5
Perfectly
31
Q

As a solicitor, which of the following would NOT be a breach of client confidentiality?

A) Updating your parents on the work you are doing for your sibling.

B) Sending details of your client’s financial circumstances to solicitors acting for the client’s buyer.

C) Sending details of your client’s property to solicitors acting for the client’s buyer.

D) Advertising on your website that you act for a famous client without disclosing the precise nature of your instructions.

E) Notifying your client’s lender (who is also your client) that the price of the property your client is buying is lower than the lender was told.

A

C) Sending details of your client’s property to solicitors acting for the client’s buyer

(Correct. You are likely to have either express (in your client care letter) or implied consent to provide this information which is necessary to progress the sale)

How well did you know this?
1
Not at all
2
3
4
5
Perfectly
32
Q

Which one of the following best describes who the SRA Accounts Rules apply to?

A) All firms (including sole practices) the SRA regulates, including all who manage or work within them.

B) Managers and employees of a law firm’s accounts department only.

C) Authorised bodies only.

D) All law firms (including sole practices) worldwide, including all who manage or work within them.

A

A) All firms (including sole practices) the SRA regulates, including all who manage or work within them

(Correct. The SRA make this clear in their introduction to the SRA Accounts rules and in Rule 1)

How well did you know this?
1
Not at all
2
3
4
5
Perfectly
33
Q

Who is responsible for compliance with the SRA Accounts Rules?

A) The managers of an authorised body are severally responsible.

B) All managers and employees of an authorised body are jointly and severally responsible.

C) Only the authorised body’s compliance manager is responsible.

D) The managers of an authorised body are jointly and severally responsible.

A

D) The managers of an authorised body are jointly and severally responsible.

(Correct. Rule 1.2 states that: ‘The authorised body’s managers are jointly and severally responsible for compliance by the authorised body, its managers and employees with these rules)

How well did you know this?
1
Not at all
2
3
4
5
Perfectly
34
Q

Which topic do the majority of the SRA Accounts Rules relate to?

A) Accountants’ reports and storage and retention of accounting records.

B) Application of the SRA Accounts Rules

C) Dealings with other money (not client money) belonging to clients or third parties.

D) Client money and client accounts

A

D) Client money and client accounts

(Correct. Rules 2-8 (of 13) are on this topic and are the largest section of the SRA Accounts Rules)

How well did you know this?
1
Not at all
2
3
4
5
Perfectly
35
Q

A client is selling a property. You receive the deposit money for the property from the buyer’s solicitor before exchange and hold it to their order, as requested.

Which one of the following best explains whether the money is client money or not?

A) The money is client money because it is held by you on behalf of a third party in relation to regulated services delivered by you.

B) The money is not client money because you do not provide regulated services to the sender of the money (the buyer’s solicitor).

C) The money is not client money because it does not belong to a client of yours.

A

A) The money is client money because it is held by you on behalf of a third party in relation to regulated services delivered by you

(Correct. This situation falls within Rule 2.1(b))

How well did you know this?
1
Not at all
2
3
4
5
Perfectly
36
Q

Your client is buying a property and prior to completion of the purchase sends you a cheque in respect of the stamp duty land tax which will be paid on the purchase after completion.

Which one of the following best explains what kind of money this cheque represents and why?

A) It is non-client money as a bill has not yet been delivered to the client for the stamp duty land tax.

B) It is non-client money as the stamp duty land tax is a paid disbursement.

C) It is client money because it has been received in respect of an unpaid cost or expense.

A

C) It is client money because it has been received in respect of an unpaid cost or expense.

(Correct. It is client money under Rule 2.1(d))

How well did you know this?
1
Not at all
2
3
4
5
Perfectly
37
Q

Which one of the following is correct in respect of non-client money?

A) If you receive money from a client in respect of a paid disbursement, it is non-client money.

B) If you receive money generally on account of costs in a transaction from a client, it will be non-client money as it is not for a specific cost.

C) If you receive money from a client in respect of an unpaid disbursement, it is non-client money

A

A) If you receive money from a client in respect of a paid disbursement, it is non-client money

(Correct. It will fall outside Rule 2.1(d) and the definition set out there of client money and so it will be non-client money)

How well did you know this?
1
Not at all
2
3
4
5
Perfectly
38
Q

Which one of the following is NOT a requirement under Rule 8 of the SRA Accounts Rules?

A) To keep accurate, contemporaneous and chronological records in respect of client ledgers (including the business side of the client ledger account) and a cash book for client accounts.

B) To, at least every five weeks, for all client accounts, reconcile the bank statement with the cash book balance and the client ledger total.

C) To keep a record of all bills or other notifications of costs given by you.

D) To obtain, at least every week, bank statements for all client accounts and business accounts which you hold or operate

A

D) To obtain, at least every week, bank statements for all client accounts and business accounts which you hold or operate

(Correct. Whilst this looks like the requirement under Rule 8.2, the correct frequency in Rule 8.2 is at least every five weeks not at least every week. The time period (at least every five weeks) matches that of the reconciliation required in Rule 8.3)

How well did you know this?
1
Not at all
2
3
4
5
Perfectly
39
Q

Which of the following correctly sets out the accounts where a receipt of client money from Dr Jones (a client) on account of costs for her property purchase would be recorded by the authorised body?

A) Business side of the client ledger for Dr Jones’ property purchase and the client cash book.

B) Client side of the client ledger for Dr Jones’ property purchase and the business account cash book.

C) Business side of the client ledger for Dr Jones’ company incorporation and the client cash book.

D) Client side of the client ledger for Dr Jones’ property purchase and the client cash book

A

D) Client side of the client ledger for Dr Jones’ property purchase and the client cash book.

(Correct. See Rules 8.1(a)(i) and 8.1(c) as well as Rule 8.1(a) and SRA guidance ‘Helping you keep accurate accounting records’)

How well did you know this?
1
Not at all
2
3
4
5
Perfectly
40
Q

Which one of the following statements about the client ledger balance is correct?

A) The balance on the client side of a client’s client ledger account shows how much money there is in total in the client account.

B) The balance on the client side of a client’s client ledger account shows how much money in the client account belongs to that client.

C) The balance on the client side of a client’s client ledger account shows how much money that client owes to the authorised body.

A

B) The balance on the client side of a client’s client ledger account shows how much money in the client account belongs to that client.

How well did you know this?
1
Not at all
2
3
4
5
Perfectly
41
Q

C wishes to bring a personal injury claim against a retailer following an accident in a shop. The claim is valued at £75,000. What is the best advice to give C about whether to issue a claim in the County Court or the High Court?

A) The claim can be issued in either the County Court or the High Court.

B) The claim should be issued in the Chancery Division of the High Court.

C) The claim must be issued in the County Court.

D) The claim must be issued in the High Court.

E) The claim should be issued in a specialist division of the County Court

A

A) The claim can be issued in either the County Court or the High Court

(Correct. As this is a personal injury claim exceeding £50,000, both the County Court and High Court have jurisdiction. You will need to consider the factors of the value, complexity and importance to the public when making a final decision about whether to issue in the County Court or the High Court)

How well did you know this?
1
Not at all
2
3
4
5
Perfectly
42
Q

If the court has ordered that one party will pay the costs of another party, the court will also need to put a figure on those costs. When it is deciding this figure, it will generally only allow recovery of…

A) …a reasonable and proportionate sum.

B) …a sum which is proportionate to the value of the claim.

C) …a reasonable sum.

A

A) …a reasonable and proportionate sum.

How well did you know this?
1
Not at all
2
3
4
5
Perfectly
43
Q

A wholesaler brings a breach of contract claim against a manufacturer. The wholesaler incurs costs of £85,000 in bringing this claim. What is the best advice to give to the wholesaler in relation to the cost consequences of the wholesaler succeeding at trial?

A) The wholesaler is likely to recover £85,000 in relation to costs.

B) The wholesaler will recover £85,000 in relation to costs.

C) The wholesaler is likely to be awarded its costs, but not the full £85,000.

D) The wholesaler may recover its costs, and they will be assessed at less than £85,000.

A

C) The wholesaler is likely to be awarded its costs, but not the full £85,000

(Correct. The general rule indicates that the wholesaler should recover its costs, so this is the ‘likely’ outcome on the facts presented, but the court will assess these costs, so it is unlikely that the full £85,000 will be recovered)

How well did you know this?
1
Not at all
2
3
4
5
Perfectly
44
Q

The general rule in relation to costs is that…

A) The party paying costs is stipulated in the CPR.

B) Each party will bear its own costs.

C) The court has a discretion to determine whether one party will pay the other party’s costs.

D) The unsuccessful party will be ordered to pay the costs of the successful party.

A

D) The unsuccessful party will be ordered to pay the costs of the successful party.

(This is the general rule. In terms of the other answers, it is correct that the court has a discretion to determine whether one party will pay the other party’s costs, but this is not what you would describe as a ‘general rule’. It is very occasionally true that the party paying costs is stipulated in the CPR, but this is the exception rather than the rule)

How well did you know this?
1
Not at all
2
3
4
5
Perfectly
45
Q

*Which one of the following would be a binding precedent in the Magistrates Court?**

A) A decision of the Crown Court

B) A dissenting judgment in the High Court

C) A decision of the Court of Appeal

D) A decision of the County Court.

E) An obiter dicta in a decision of the Supreme Court

A

C) A decision of the Court of Appeal

How well did you know this?
1
Not at all
2
3
4
5
Perfectly
46
Q

Which one of the following statements is incorrect?

A) The Supreme Court in its Practice Statement of 1966 recognises too rigid adherence to precedent may lead to injustice and unduly restrict the proper development of the law.

B) The Supreme Court in departing from a previous judgement will bear in mind the danger of disturbing retrospectively the basis on which contract, settlement of property and fiscal arrangements have been entered into and the need for certainty as to the criminal law.

C) The Supreme Court is bound by its previous decisions pursuant to the Practice Statement 1966 and will only depart from a previous decision when it appears right to do so.

D) The Supreme Court was bound by its previous decisions before the Practice Statement of 1966. It sees the use of precedent as an indispensable foundation which provides legal certainty and allows the orderly development of legal rules.

E) The Supreme Court is not bound by its previous decisions pursuant to the Practice Statement 1966 and will only depart from a previous decision when it appears right to do so.

A

C) The Supreme Court is bound by its previous decisions pursuant to the Practice Statement 1966 and will only depart from a previous decision when it appears right to do so

(Correct. This is incorrect. A Supreme Court by virtue of the Practice Statement 1966 is not bound by its previous decisions. It rarely will depart and will do so when it appears right to do so)

How well did you know this?
1
Not at all
2
3
4
5
Perfectly
47
Q

In which one of the following circumstances will the Court of Appeal be bound by a previous Court of Appeal judgment on the same issue?

A) Where the Court of Appeal considers an earlier Court of Appeal decision to have been given per incuriam.

B) Where the Court of Appeal considers it would be in the interests of justice to depart from an earlier Court of Appeal decision.

C) Where the Court of Appeal is faced with two earlier conflicting Court of Appeal decisions

D) Where the Court of Appeal decision conflicts with a Supreme Court Decision

E) Where the Court of Appeal does not agree with an earlier Court of Appeal decision

A

E) Where the Court of Appeal does not agree with an earlier Court of Appeal decision

(Correct. This answer is incorrect. Young v Bristol Aeroplane sets out the three circumstances where a Court of Appeal can depart from a previous Court of Appeal judgement on the same issue)

How well did you know this?
1
Not at all
2
3
4
5
Perfectly
48
Q

_________categories of civil claim qualify in principle for Civil CLS funding.

A) No

B) Very few

C) Most

D) All

A

B) Very few

(Correct. The vast majority of civil legal work is not eligible for CLS funding)

How well did you know this?
1
Not at all
2
3
4
5
Perfectly
49
Q

Which of the following best describes the situation in relation to a CFA where the client wins their case at trial?

A) The client may recover the basic fee, disbursements, damages and success fee from the opponent

B) The client may recover the basic fee from the opponent but cannot recover the disbursements, success fee or damages

C) The client may recover the basic fee and disbursements from the other side, together with damages. The client will owe the solicitor a success fee which will be payable from the damages and is not recoverable from the opponent

D) The client may recover the basic fee and damages from the opponent. The client will owe the solicitor a success fee which will be payable from the damages and is not recoverable from the opponent.

A

C) The client may recover the basic fee and disbursements from the other side, together with damages. The client will owe the solicitor a success fee which will be payable from the damages and is not recoverable from the opponent

How well did you know this?
1
Not at all
2
3
4
5
Perfectly
50
Q

Civil CLS funding is subject to ____________

A) A merit test.

B) A means test.

C) A means test and a merit test

D) A public interest test.

E) A merit test and a public interest test.

A

C) A means test and a merit test

(Correct. Civil CLS funding is only available for clients with limited income and capital, and if the client’s position is sufficiently meritorious)

How well did you know this?
1
Not at all
2
3
4
5
Perfectly
51
Q

Your client, Mr. Ibbotson, would like to know how damages based agreements (‘DBAs’) work in practice. Which one of the following best describes the advice you would give to him on this matter in relation to a typical DBA?

A) Lawyers do not recover any fees if they lose the case. If they win, they receive a proportion of the damages awarded to the client by way of payment for their work.

B) Lawyers will recover their basic fee if they lose the case. If they win, they can recover a success fee from the damages awarded to the client.

C) Lawyers do not recover any fees if they lose the case. If they win, they can recover their normal fees plus an uplift out of the damages awarded to the client.

D) Lawyers will recover their basic fee if they lose the case. If they win, they can recover double the basic fee.

A

A) Lawyers do not recover any fees if they lose the case. If they win, they receive a proportion of the damages awarded to the client by way of payment for their work.

How well did you know this?
1
Not at all
2
3
4
5
Perfectly
52
Q

Mrs Meade has been told that her solicitors have taken on her claim on a ‘no win, no fee’ CFA basis. Does this mean that if she loses, she won’t have to pay any costs?

A) No. She will still be liable for her own disbursements and might still be held liable for the other side’s costs.

B) Yes. A conditional fee agreement means that she won’t have to pay any solicitors’ costs if she loses.

C) Yes. Her solicitors will have to pay the defendant’s costs if she loses.

D) No. She will still be liable for both sides’ costs.

A

A) No. She will still be liable for her own disbursements and might still be held liable for the other side’s costs.

How well did you know this?
1
Not at all
2
3
4
5
Perfectly
53
Q

Martha Van Hoffer is the managing director of VanH Limited (“VanH”). VanH have instructed your firm to act on their behalf in relation to a large litigation action. Martha is interested in taking out ‘after the event’ insurance, if possible, but is concerned about the level of the premium. She would like to know whether any premium might be recoverable from her opponent if VanH is successful in the litigation. VanH do not have ‘before the event’ legal expenses insurance.

Which one of the following statements is correct if VanH win at trial and is awarded its costs?

A) VanH will not be able to claim the premium.

B) If successful, VanH should be able to claim any premium paid from the other side as part of the costs of the proceedings, so long as it is reasonable

C) VanH will only be able to claim a certain percentage of the premium, provided that it is not in excess of £10,000

D) VanH is not in fact able to take out ‘after the event’ insurance because it is a company, not an individual

A

A) VanH will not be able to claim the premium

(Correct. Well done. ATE insurance premiums are not recoverable from an opponent)

How well did you know this?
1
Not at all
2
3
4
5
Perfectly
54
Q

Which ONE of the following is the correct definition of an undertaking as set out in the SRA Glossary?

A) “A statement, given orally or in writing, whether or not it includes the word “undertake” or “undertaking” to someone who reasonably places reliance on it, that you or a third party will do something or cause something to be done, or refrain from doing something”.

B) “A statement, given in writing to someone that you or a third party will do something or cause something to be done, or refrain from doing something”.

C) “A statement, given orally or in writing to someone who reasonably places reliance on it, that you or a third party will do something or cause something to be done, or refrain from doing something”

A

A) “A statement, given orally or in writing, whether or not it includes the word “undertake” or “undertaking” to someone who reasonably places reliance on it, that you or a third party will do something or cause something to be done, or refrain from doing something”

(Correct. Note that
an undertaking can be given orally or in writing, and
does not have to include the word “undertake” or “undertaking”.
If the recipient reasonable places reliance on the statement it will be binding on the person giving it)

How well did you know this?
1
Not at all
2
3
4
5
Perfectly
55
Q

Which ONE of the following statements about undertakings is CORRECT?

A) An undertaking can be given by any individual in a law firm who is legally qualified. An undertaking could therefore be given by a partner or a qualified lawyer, but not by an apprentice, or a secretary.

B) An undertaking can only be given by a partner in a law firm.

C) An undertaking can be given by any individual in a law firm provided it can be reasonably relied upon by the recipient. An undertaking could therefore be given by a partner, a qualified lawyer, an apprentice, secretary or any other individual in the firm

A

C) An undertaking can be given by any individual in a law firm provided it can be reasonably relied upon by the recipient. An undertaking could therefore be given by a partner, a qualified lawyer, an apprentice, secretary or any other individual in the firm

(Correct. Note that it is important that any individual in a law firm can be found to have given an undertaking)

How well did you know this?
1
Not at all
2
3
4
5
Perfectly
56
Q

Which provision of CCS contains the following requirement:

“You draw the court’s attention to relevant cases and statutory provisions, or procedural irregularities of which you are aware, and which are likely to have a material effect on the outcome of the proceedings.”

A) CCS 2.3

B) CCS 2.4

C) CCS 2.7

D) CCS 2.6

A

C) CCS 2.7

(Correct. CCS 2.3 contains the requirement that you do not provide or offer to provide any benefit to witnesses dependent upon the nature of their evidence or the outcome of the case. CCS 2.4 contains the requirement that you only make assertions or put forward statements, representations or submissions to the court or others which are properly arguable. CCS 2.6 contains the statement that you do not waste the court’s time)

How well did you know this?
1
Not at all
2
3
4
5
Perfectly
57
Q

Which one of the following is not a quality you would expect from a legal professional?

A) A desire to get the best outcome for your client, whatever the circumstances.

B) A high level of education and training.

C) A desire to promote the public good.

D) Morality.

E) Integrity.

A

A) A desire to get the best outcome for your client, whatever the circumstances

(Correct. The circumstances may be such that your obligation to act in the best interest of your client is replaced by an overriding duty to promote the public good)

How well did you know this?
1
Not at all
2
3
4
5
Perfectly
58
Q

Which one of the following statements correctly describes the key aspects of being a legal professional?

A) Being a legal professional requires you to demonstrate knowledge, skill, and expertise and therefore a high level of education and training. Provided you act in the best interests of your client, you will meet the standard expected of a legal professional.

B) You do not need to worry about demonstrating trust, morality and integrity if you are working in-house because your only obligation is to present the best case for your employer.

C) As well as knowledge and expertise, being a legal professional requires you to demonstrate trust, morality and integrity. Trust, morality and integrity are important so that your clients know that they can trust the advice you give them.

D) As well as knowledge and expertise, being a legal professional requires you to demonstrate trust, morality and integrity. Trust, morality and integrity are important to maintain the public’s trust in the legal profession.

A

D) As well as knowledge and expertise, being a legal professional requires you to demonstrate trust, morality and integrity. Trust, morality and integrity are important to maintain the public’s trust in the legal profession.

(Correct. As well as owing a duty to your clients, you also have a duty to promote the public good. Trust, morality and integrity are therefore important qualities in a legal professional)

How well did you know this?
1
Not at all
2
3
4
5
Perfectly
59
Q

Your colleague, a newly qualified solicitor in the property department, is giving you a lift back to the office after a football match. Your colleague has a minor collision with another car. As your colleague exchanges details with the owner of the other car, you hear your colleague say:

“Don’t bother suing me for the damage to your car. I’ve just qualified as a solicitor so you will be wasting your time. It’ll cost you so much in legal costs, which you won’t recover whatever happens, I’m sure you’ll regret suing me for a very long time”.

Which one of the following statements is true?

A) Your colleague has not breached the standards expected of a legal professional because the collision was not with a client.

B) Your colleague has breached the standards expected of a legal professional.

C) Your colleague is giving free advice to the owner of the other car and has not breached the standards expected of a legal professional.

D) Your colleague has not breached the standards expected of a legal professional because the collision occurred outside working hours.

A

B) Your colleague has breached the standards expected of a legal professional

(Correct. The standards of truth, integrity and morality apply to legal professionals in their personal as well as their professional lives and when they are dealing with members of the public as well as clients)

How well did you know this?
1
Not at all
2
3
4
5
Perfectly
60
Q

Which one of the following is not a power of the Solicitors Disciplinary Tribunal?

A) Award costs against a party to proceedings.

B) Award compensation to a party that has suffered loss.

C) Impose a fine.

D) Impose a restriction on the area in which a solicitor can practice.

E) Reprimand a solicitor.

A

B) Award compensation to a party that has suffered loss

(Correct. This action would have to be taken by a court)

How well did you know this?
1
Not at all
2
3
4
5
Perfectly
61
Q

Which one of the following statements is false?

A) The Legal Services Board regulates all lawyers in England and Wales.

B) The Solicitors Disciplinary Tribunal deals with serious breaches of the SRA’s rules and disciplines solicitors.

C) Clients can refer complaints about poor service from a solicitor to the Legal Ombudsman at any time.

D) The SRA sets the principles and codes of conduct that solicitors and lawyers regulated by the SRA must comply with.

A

C) Clients can refer complaints about poor service from a solicitor to the Legal Ombudsman at any time.

(Correct. The statement is false. The right to refer a complaint to the Legal Ombudsman applies to individuals, small businesses, charities, clubs and trusts. It does not apply to larger companies. In addition the client should exhaust the law firm’s procedure before referring a complaint to the legal Ombudsman)

How well did you know this?
1
Not at all
2
3
4
5
Perfectly
62
Q

Which one of the following is not a power of the SRA?

A) Commence disciplinary proceedings against a solicitor by the Solicitors Disciplinary Tribunal

B) Order a solicitor to repay or refund the whole of part of the costs to the client.

C) Strike a solicitor off the roll.

D) Close down a law firm.

E) Impose a disciplinary sanction on a firm, for example a fine.

A

C) Strike a solicitor off the roll

(Correct. This a power of the Solicitors Disciplinary Tribunal)

How well did you know this?
1
Not at all
2
3
4
5
Perfectly
63
Q

Which one of the following is not an SRA Principle?

A) Act in a way that encourages equality, diversity and inclusion.

B) Act in a way that upholds the constitutional principle of the rule of law, and the proper administration of justice.

C) Act in the best interests of your employer.

D) Act in a way that upholds public trust and confidence in the solicitors’ profession and in legal services provided by authorised persons.

E) Act with independence.

A

C) Act in the best interests of your employer

(Correct. Acting in the best interests of your employer is not an SRA Principle. Whether you work for a law firm or in-house, you are subject to the Principles to act with independence, with honesty, with integrity, and in a way that upholds public trust and confidence in the profession. If these Principles came into conflict with a desire to act in the best interests of your employer, the Principles would take priority)

How well did you know this?
1
Not at all
2
3
4
5
Perfectly
64
Q

CCS 2.3 relates to proceedings before the courts. It states:

‘You do not provide or offer to provide any benefit to witnesses dependent upon the nature of their evidence or the outcome of the case.’

If you breached this provision of CCS, which one of the following Principles would you also be in breach of?

A) Principle 6: act in a way that encourages equality, diversity and inclusion.

B) Principle 1: act in a way that upholds the constitutional principle of the rule of law, and the proper administration of justice.

C) Principle 7: act in the best interests of each client.

A

B) Principle 1: act in a way that upholds the constitutional principle of the rule of law, and the proper administration of justice

(Correct. If you provided a benefit to witness for providing evidence of a particular nature you would be jeopardising the proper administration of justice)

How well did you know this?
1
Not at all
2
3
4
5
Perfectly
65
Q

Which ONE of the following statements about the Principles is FALSE?

A) If you behave in an untrustworthy way but that behaviour is outside of your usual professional practice, you cannot be found liable to have breached Principles 4 or 5.

B) Your duty to act with integrity (under Principle 5) extends not only to your relationship with your client but also to your dealings with the court, other lawyers and members of the public.

C) The Principles apply to all individuals authorised by the SRA to provide legal services, as well as authorised firms and their managers and employees.

D) Should the Principles come into conflict, those which safeguard the wider public interest take precedence over an individual client’s interests.

A

A) If you behave in an untrustworthy way but that behaviour is outside of your usual professional practice, you cannot be found liable to have breached Principles 4 or 5.

(Correct. Principles 4 and 5 apply to legal professionals in their personal as well as their professional lives. Therefore, the statement that you cannot be found liable to have breached Principles 4 and 5 if you behave in an untrustworthy way outside of your usual professional practice is FALSE)

How well did you know this?
1
Not at all
2
3
4
5
Perfectly
66
Q

Your supervisor has been asked to act for the local conservative MP on the sale of their house. Your supervisor is a member of the labour party and refuses to act on the sale because they do not want to ‘help’ a member of the conservative party. Which section of CCS has your supervisor breached?

A) CCS 1.1: ‘you do not unfairly discriminate by allowing your personal views to affect your professional relationships and the way in which you provide your services. ‘

B) CCS 1.4: ‘you do not mislead or attempt to mislead your clients, the court or others, either by your own acts or omissions or allowing or being complicit in the acts or omissions of others (including your client).’

C) CCS 3.2 ‘you ensure that the service you provide to clients is competent and delivered in a timely manner.’

D) CCS 1.2: ‘you do not abuse your position by taking unfair advantage of clients or others.’

A

A) CCS 1.1: ‘you do not unfairly discriminate by allowing your personal views to affect your professional relationships and the way in which you provide your services. ‘

(Correct. In refusing to accept the instructions, the supervisor has allowed their personal political views to affect their professional relationships)

How well did you know this?
1
Not at all
2
3
4
5
Perfectly
67
Q

Your client has asked you to lie to the police and provide a witness statement stating that you were in a meeting with your client at the time a crime was alleged to have been committed. You were not in a meeting with the client at that time. Should you:

A) Refuse to provide the witness statement, as to do so would breach Principles 1, 2, 3, 4 and 5.

B) Comply with the client’s request in order to comply with Principles 1 and 2.

C) Refuse to comply with the client’s request as to do so would breach Principle 6.

D) Comply with the client’s request and provide the witness statement in order to comply with Principle 7.

A

A) Refuse to provide the witness statement, as to do so would breach Principles 1, 2, 3, 4 and 5

(Correct. If you complied with your client’s instruction, you would be breaching Principles 1 (upholding the rule of law and the proper administration of justice), 2 (upholding public trust and confidence in the solicitor’s profession), 3 (act with independence), 4 (act with integrity) and 5 (act with honesty))

How well did you know this?
1
Not at all
2
3
4
5
Perfectly
68
Q

You are acting for a client in relation to a dispute. You receive a defence from the individual on the other side which is badly drafted and contains numerous errors. You suspect that the individual does not have legal representation.

Which ONE of the following statements is CORRECT?

A) You should contact the individual on the other side and suggest that they instruct a lawyer. Failing to do so puts you at risk of breaching CCS 1.1.

B) You should contact the individual on the other side and suggest that they instruct a lawyer. Failing to do so puts you at risk of breaching CCS 1.2.

C) The existence of errors in the other side’s defence is good news for your client. You should continue your client’s case as aggressively as you can in order to comply with Principle 7.

D) The existence of errors in the other side’s defence is good news for your client. You should continue your client’s case as aggressively as you can in order to comply with Principle 3.

A

B) You should contact the individual on the other side and suggest that they instruct a lawyer. Failing to do so puts you at risk of breaching CCS 1.2

(Correct. Acting for your client where the individual on the other side does not have legal representation can be very challenging and puts you at risk of breaching CCS 1.2 ‘you do not abuse your position by taking unfair advantage of clients or others’. The simplest solution for you is for the individual on the other side to get their own lawyer)

How well did you know this?
1
Not at all
2
3
4
5
Perfectly
69
Q

The SRA’s requirements in relation to equality, diversity and inclusion apply to which ONE of the following categories:

A) Legal professionals in their dealings with clients only.

B) Managers of law firms only.

C) All individuals regulated by the SRA in relation to their dealings with clients, the people they work with and other members of the public they deal with.

D) Legal Professionals in their dealings with fellow members of staff only.

A

C) All individuals regulated by the SRA in relation to their dealings with clients, the people they work with and other members of the public they deal with.

(Correct. Principle 6 is broad. As stated in the SRA’s guidance on their approach to equality, diversity and inclusion (‘EDI’), it applies to an individual’s dealings with the people they work with, clients and others)

How well did you know this?
1
Not at all
2
3
4
5
Perfectly
70
Q

Which ONE of the following is NOT a protected characteristic under the Equality Act 2010?

A) Vegetarianism

B) Age.

C) Gender Reassignment.

D) Disability.

A

A) Vegetarianism

(Correct. Vegetarianism is not one of the protected characteristics listed in your materials under the Equality Act 2010)

How well did you know this?
1
Not at all
2
3
4
5
Perfectly
71
Q

Which one of the following statements about a legal professional’s obligations in relation to EDI under the SRA’s Guidance on their approach to EDI is FALSE?

A) Law firms must collect, report and publish data about the diversity of their workforce.

B) Individuals regulated by the SRA are responsible for upholding the reputation of the profession in their professional and personal life and for treating people fairly and with dignity and respect. They should ensure their personal views do not have a negative impact on others. This includes expressing extreme personal, moral or political opinions on social media.

C) Law firms have a legal obligation to provide reasonable adjustments to disabled clients and employees to make sure they are not at a substantial disadvantage compared to those who are not disabled. Law firms may pass on the costs of those adjustments to those clients or employees.

D) Firms and individuals regulated by the SRA must encourage diversity at all levels of the workforce.

A

C) Law firms have a legal obligation to provide reasonable adjustments to disabled clients and employees to make sure they are not at a substantial disadvantage compared to those who are not disabled. Law firms may pass on the costs of those adjustments to those clients or employees.

(Correct. This statement is FALSE. Read the guidance note again to check whether law firms can pass on the costs of such adjustments)

How well did you know this?
1
Not at all
2
3
4
5
Perfectly
72
Q

Which ONE of the following is NOT a protected characteristic under the Equality Act 2010?

A) Measles

B) Race

C) Sex

D) Religion or belief

A

A) Measles

(Correct. Measles is not one of the protected characteristics listed in your materials under the Equality Act 2010)

How well did you know this?
1
Not at all
2
3
4
5
Perfectly
73
Q

Which one of the following is NOT an example of direct discrimination?

A) A shop will not serve someone because of their ethnic origin.

B) A nightclub charges a higher price for entry to a man because of their sex where the service provided to a woman is exactly the same.

C) A bowls club will not admit someone with a hearing impairment as a member.

D) A large department store provides a private resting area for women who are pregnant or breastfeeding. A man feeling tired complains that he does not have access to a similar facility.

A

D) A large department store provides a private resting area for women who are pregnant or breastfeeding. A man feeling tired complains that he does not have access to a similar facility.

(Under section 13(1) Equality Act, direct discrimination occurs where “because of a protected characteristic, A treats B less favourably than A treats or would treat others”.
Note however that, under section 13(6)(b) of the Act, in considering discrimination against a man, it is not relevant to take into account any special treatment given to a woman in connection with pregnancy or childbirth. This example is not discrimination because a man cannot compare himself to a woman in connection with pregnancy, childbirth, or it would follow, breast feeding.
The other examples would amount to direct discrimination:
A shop will not serve someone because of their ethnic origin: in this example the protected characteristic is race and the shop is treating the customer less favourably than they would treat others.
A nightclub charges a higher price for entry to a man because of their sex where the service provided to a woman is exactly the same: in this example the protected characteristic is sex. The night club owner is treating men less favourably than they treat others (i.e women).
A bowls club will not admit someone with a hearing impairment as a member: in this example the protected characteristic is disability. The bowls club is treating the person with a hearing impairment less favourably than they treat others (people without a hearing impairment))

How well did you know this?
1
Not at all
2
3
4
5
Perfectly
74
Q

Which ONE of the following statements about the duty to make reasonable adjustments is CORRECT?

A) The duty to make reasonable adjustments aims to make sure that a disabled person can use a service as close as it is reasonably possible to get to the standard usually offered to non-disabled people.
If a disabled person wants to use the service, the service provider must ensure that it makes reasonable adjustments to the service within a reasonable time of the disabled person’s request.
Many adjustments will not be particularly expensive and a service-provider is not required to do more than is reasonable.

B) The duty to make reasonable adjustments aims to make sure that a disabled person can use a service as close as possible to get to the standard usually offered to non-disabled people.
The duty is ‘anticipatory’. This means a service provider cannot wait until a disabled person wants to use the service. They must think in advance (and on an ongoing basis) about what disabled people with a range of impairments (such as people who have a visual or hearing impairment, a mobility impairment or a learning disability) might reasonably need.
The service-provider must use its best endeavours to provide the same service to a disabled person as to a non-disabled person, and bear the cost of all necessary expenditure.

C) The duty to make reasonable adjustments aims to make sure that a disabled person can use a service as close as it is reasonably possible to get to the standard usually offered to non-disabled people.
The duty is ‘anticipatory’. This means a service provider cannot wait until a disabled person wants to use the service. They must think in advance (and on an ongoing basis) about what disabled people with a range of impairments (such as people who have a visual or hearing impairment, a mobility impairment or a learning disability) might reasonably need.
Many adjustments will not be particularly expensive and a service-provider is not required to do more than is reasonable.

A

C) The duty to make reasonable adjustments aims to make sure that a disabled person can use a service as close as it is reasonably possible to get to the standard usually offered to non-disabled people.
The duty is ‘anticipatory’. This means a service provider cannot wait until a disabled person wants to use the service. They must think in advance (and on an ongoing basis) about what disabled people with a range of impairments (such as people who have a visual or hearing impairment, a mobility impairment or a learning disability) might reasonably need.
Many adjustments will not be particularly expensive and a service-provider is not required to do more than is reasonable

How well did you know this?
1
Not at all
2
3
4
5
Perfectly
75
Q

You have been helping your supervisor write advice to a client on the likelihood of his claim against a counter party succeeding. You have just amended the advice and your supervisor has not yet reviewed it. As your supervisor left the office for the evening, she asked you to email it to her that evening so that she could review it before sending it to the client tomorrow morning.

The client rings you up. He has been trying to get hold of your supervisor. The client explains that the advice has suddenly become urgent and asks you to email it to him as soon as possible that evening so that he can send it to his managing director who is waiting for it.

Which one of the following should you do?

A) Explain the position to the client. Say that you will try to get hold of your supervisor and get back to the client. Then leave for the evening. You have a team drinks event you have been looking forward to. The matter can wait until the morning.

B) Refuse to send the advice to the client. Tell him that he will have to wait until your supervisor is back in the morning. Then leave the office for the evening, you have a team drinks event you have been looking forward to.

C) Email the advice to the client straight away. It is nearly final, your supervisor seemed happy with it and you feel relatively confident that your supervisor would be happy for you to send the advice out.

D) Explain the position to the client. Say that you will try to get hold of your supervisor and check whether it is ok to send him the advice. Then ring your supervisor on her mobile, follow her instructions and ring the client back and update him.

A

D) Explain the position to the client. Say that you will try to get hold of your supervisor and check whether it is ok to send him the advice. Then ring your supervisor on her mobile, follow her instructions and ring the client back and update him.

(Correct. This course of action would be the best. You should not send the advice out without your supervisor’s approval. There may be points that she wants to change. In getting your supervisor to check the work first and, at the same time, keeping your client informed of what is going on, you are providing a competent service to your client in a timely manner in accordance with CCS 3.2)

How well did you know this?
1
Not at all
2
3
4
5
Perfectly
76
Q

Which paragraph of the SRA Code of Conduct for Solicitors, RELs and RFLs (‘CCS’) contains the provision that you must not seek to influence the substance of evidence, including generating false evidence or persuading witnesses to change their evidence?

A) CCS 2.2

B) CCS 2.1

C) CCS 1.4

A

A) CCS 2.2

(Correct. CCS 2.1 states that you must not misuse or tamper with evidence, and CCS 1.4 contains the rule that you must not mislead your clients, the court or other people)

How well did you know this?
1
Not at all
2
3
4
5
Perfectly
77
Q

You are a solicitor in the residential conveyancing department. Your client, the seller, is due to complete on a sale today. You received part of the completion monies (£450,000) from the buyer’s solicitors yesterday, which you undertook to hold to the order of the buyer’s solicitors pending completion.

You have just received an email from the buyer’s solicitors advising you that their clients have instructed them not to proceed with the completion today as their mortgage advance will not be available for another four days. The buyer’s solicitors have instructed you to return the £450,000 immediately. You have (correctly) explained to your client that by failing to complete today, the buyer is now in breach of contract.

Your client is aware that you hold the £450,000 and has instructed you to send him the money immediately.

Which ONE of the following statements CORRECTLY states what you should do next?

A) You should not return the money to the buyer’s solicitors because the buyer is in breach of contract. You should not forward the money to your client because you would be breaching your undertaking to the buyer’s solicitors. You should therefore keep the money in your firm’s account until the buyer has sufficient funds to complete the transaction.

B) You must return the £450,000 to the buyer’s solicitors immediately, even though their client is in breach of contract. If you fail to do so you will have breached your undertaking to the buyer’s solicitors.

C) You should forward the £450,000 to your client, as part payment of the balance due to your client.

A

B) You must return the £450,000 to the buyer’s solicitors immediately, even though their client is in breach of contract. If you fail to do so you will have breached your undertaking to the buyer’s solicitors.

(Correct. You undertook to hold the £450,000 to the order of the buyer’s solicitors. This means that you cannot send the money to your client without the agreement of the buyer’s solicitors. The money does not belong to you or to your client. If you fail to return the money, you will be in breach of CCS 1.3.)

How well did you know this?
1
Not at all
2
3
4
5
Perfectly
78
Q

Which ONE of the following statements completes the following sentence CORRECTLY?

“Section 56(1) of the Legal Aid, Sentencing and Punishment of Offenders Act 2012 (‘LASPO’) prohibits …

A) … solicitors from referring or receiving referrals in respect of a claimant’s claim for damages in respect of personal injury or death to each other in consideration for a referral fee.

B) … solicitors from referring or receiving referrals in respect of claims which are subject to legal restrictions imposed in the interests of national security

C) … solicitors from referring or receiving referrals in respect of a clients who are the subject of criminal proceedings.

A

A) … solicitors from referring or receiving referrals in respect of a claimant’s claim for damages in respect of personal injury or death to each other in consideration for a referral fee

(Correct. This is a prohibited referral fee. Solicitors are not permitted to pay or receive a fee for referring or receiving referrals for these types of claims)

How well did you know this?
1
Not at all
2
3
4
5
Perfectly
79
Q

Which provision of CCS states:

“you ensure that any publicity in relation to your practice is accurate and not misleading, including that relating to your charges and the circumstances in which interest is payable by or to clients.”

A) CCS 8.7

B) CCS 8.8

C) CCS 8.9

A

B) CCS 8.8

(CCS 8.9 contains the requirement that you do not make unsolicited approaches to members of the public in order to advertise legal services.
CCS 8.7 states that you must ensure that clients receive the best possible information about how their matter will be priced and, both at the time of engagement and when appropriate as their matter progresses, about the likely overall cost of the matter and any costs incurred)

How well did you know this?
1
Not at all
2
3
4
5
Perfectly
80
Q

Easy Property Selling Limited (‘EPS’) is a firm of estate agents. It has agreed to introduce property sellers and buyers to Price Prior’s residential property department. Price Prior has agreed to pay EPS £200 plus VAT in respect of each such introduction.

Which ONE of the following statements is CORRECT

A) This arrangement is an introduction and Price Prior will need to comply with CCS 5.1

B) This arrangement is not an introduction or a referral because the arrangement is not with a lawyer, so CCS 5.1 does not apply.

C) This arrangement relates to a referral fee which will not be permitted under CCS 5.2.

A

A) This arrangement is an introduction and Price Prior will need to comply with CCS 5.1

(Correct. This arrangement is an introduction. Price Prior is
making a payment to a third party (either within your firm or outside) for introducing the client;
the payment is a fixed fee or calculated with reference to the amount being charged to the client as a percentage of your gross or net fees; and
the third party may be another lawyer, but not necessarily so.
Here the third party is not a lawyer, but this does not stop the arrangement being caught by CCS 5.1)

How well did you know this?
1
Not at all
2
3
4
5
Perfectly
81
Q

Which ONE of the following statements is CORRECT about the following data protection principle:

‘personal data must be handled in a way that ensures appropriate security, including protection against unlawful or unauthorised processing, access, loss, destruction or damage’.

A) It applies only to law firms.

B) It applies only to partners and managers in an organisation.

C) It applies to you and everyone in your workplace.

A

C) It applies to you and everyone in your workplace.

(Correct. This principle is something everyone needs to be concerned with. Accidental disclosure / human error is the most common cause of breach of the data protection legislation. You must comply with your employer’s IT / data protection policy. For example, not disclose your password etc)

How well did you know this?
1
Not at all
2
3
4
5
Perfectly
82
Q

Which ONE of the following statements is CORRECT?

A) Personal data does not include information about a person that is public knowledge.

B) Information about someone’s professional life cannot be personal data for the purposes of the Data Protection Act.

C) Personal data’ does not include truly anonymous information. However, if you could still identify someone from the details, or by combining it with other information, it will count as personal data.

A

C) Personal data’ does not include truly anonymous information. However, if you could still identify someone from the details, or by combining it with other information, it will count as personal data.

How well did you know this?
1
Not at all
2
3
4
5
Perfectly
83
Q

You are making amendments to a witness statement you and your supervisor are finalising ready to serve at court and on the other side in a few days’ time. You receive a call from a friend who is working at the firm acting for the other side. The friend asks if you can send her the witness statement that evening so that her supervisor can read it and prepare for the hearing.

Which one of the following correctly states what you should do and why?

A) Anonymise the witness statement to remove any personal information about your client, and send it to your friend. You will be serving it on the other side soon, and a few days’ advance notice is unlikely to cause your client any harm, and at least you will be complying with the Data Protection Act.

B) Refuse your friend’s request because you would be breaching your duty of confidentiality to your client if you disclosed this information to your friend.

C) Email the draft witness statement to your friend that evening. You will be serving it on the other side soon, and a few days’ advance notice is unlikely to cause your client any harm. In addition, your friend might help you out on another matter should the occasion arise.

A

B) Refuse your friend’s request because you would be breaching your duty of confidentiality to your client if you disclosed this information to your friend.

(Correct. The witness statement contains information which is confidential to your client. You would be breaching your duty of confidentiality to your client under CCS 6.3 if you disclosed this information to any person, let alone the other side, without your client’s consent or being required to do so by law.)

How well did you know this?
1
Not at all
2
3
4
5
Perfectly
84
Q

Which ONE of the following statements is CORRECT

A) The duty of disclosure under CCS 6.4 does not apply where you have reason to believe that serious physical or mental injury will be caused to your client or another if the information is disclosed.

B) The duty of disclosure under CCS 6.4 does not apply where you are satisfied it is reasonable in all the circumstances for you not to disclose information to the client.

C) The duty of disclosure under CCS 6.4 does not apply where the Compliance Officer for Legal Practice consents to you not disclosing the information to the client.

A

A) The duty of disclosure under CCS 6.4 does not apply where you have reason to believe that serious physical or mental injury will be caused to your client or another if the information is disclosed

(Correct. This is an exception to the rule in CCS 6.4, as set out in CCS 6.4(c ).
Under CCS and CCF, the decision as to whether or not you should disclose information that is material to your client’s matter does not rest with your Compliance Officer for Legal Practice or with what you consider to be reasonable)

How well did you know this?
1
Not at all
2
3
4
5
Perfectly
85
Q

In which ONE of the following circumstances will there not have been a breach of the duty of confidentiality?

A) Disclosing information about your deceased client’s estate to a beneficiary without the consent of the deceased’s personal representatives.

B) Disclosing information about your client (A) from the firm you used to work for, which is material to the litigation matter on which you are advising your client (B) at your new firm.

C) Providing information to the court in compliance with a court order.

A

C) Providing information to the court in compliance with a court order.

(Correct. Complying with a court order falls within the exception “unless disclosure is required…by law” under CCS 6.3)

How well did you know this?
1
Not at all
2
3
4
5
Perfectly
86
Q

Which provision of the Code of Conduct for Solicitors, RELs and RFLs (‘CCS’) contains the duty of confidentiality:

“You keep the affairs of current and former clients confidential unless disclosure is required or permitted by law or the client consents”

A) CCS 6.2

B) CCS 6.3

C) CCS 6.4

A

B) CCS 6.3

(CCS 6.2 contains the principle that you do not act if you have a conflict of interest or a significant risk of a conflict of interest.
CCS 6.4 contains the principle that where you are acting for a client on a matter, you make the client aware of all information material to the matter of which you have knowledge (the duty of disclosure))

How well did you know this?
1
Not at all
2
3
4
5
Perfectly
87
Q

Which ONE of the following statements is correct?

A) The 3 Cs are customer due diligence, conflict check and client care letter

B) The 3 Cs are conflict check, confidentiality and competence.

C) The 3 Cs are customer due diligence, confidentiality and client care letter.

A

A) The 3 Cs are customer due diligence, conflict check and client care letter

88
Q

Which ONE of the following statements is correct?

A) CDD is verifying the identity of your client on the basis of information from a government source.

B) CDD is identifying your client by taking a copy of their passport.

C) CDD is identifying your client and verifying their identity on the basis of documents, data or information from a reliable source

A

C) CDD is identifying your client and verifying their identity on the basis of documents, data or information from a reliable source

89
Q

Which provision of CCS contains the principle that you do not act in relation to a matter if you have a conflict of interest?

A) CCS 6.3

B) CCS 6.2

C) CCS 6.1

A

B) CCS 6.2

90
Q

In which ONE of the following scenarios should you NOT accept the client’s instructions?

A) If you consider the client’s instructions to be contrary to your religious belief.

B) If you would be breaching the law in accepting the client’s instructions.

C) If you do not like the client.

A

B) If you would be breaching the law in accepting the client’s instructions

(Correct. Under Principle 1 you must act in a way that upholds the constitutional principle of the rule of law, and the proper administration of justice. You would not be able to comply with this Principle if you accepted instructions that require you to breach the law.
The other options may put you in a difficult position personally and there might be an argument for getting a colleague to act on the matter, but it is not a reason to refuse to act at all)

91
Q

You are an apprentice in the litigation department. You have been working long hours preparing for a case that is coming to trial next week. You are preparing a document your supervisor has asked you to complete so that it can be sent out to the client that day. You think it will take you about three hours to prepare the document. You also think you are likely to be working at the weekend preparing for the trial. You receive a telephone call from a different client whom you have advised previously, asking for your help on a new matter. The new client wants you to attend a conference call from 3pm until 5pm that afternoon. What should you do?

A) Advise the new client that although you may not personally be able to attend the call that afternoon, you will speak to your supervisor to see if you or another lawyer in your team can attend. Then speak to your supervisor and agree whether you should work on the document for the trial or attend the call and advise the client accordingly.

B) Explain to the new client that unfortunately you are very busy at the moment so will not be able to attend the conference call, and recommend they try a different law firm.

C) Agree to attend the call with the new client. You want to do the best for that client and build up a good relationship with them. You will worry about the trial later.

D) Explain to the client that you cannot attend a call that afternoon and agree to attend a conference call tomorrow morning instead.

A

A) Advise the new client that although you may not personally be able to attend the call that afternoon, you will speak to your supervisor to see if you or another lawyer in your team can attend. Then speak to your supervisor and agree whether you should work on the document for the trial or attend the call and advise the client accordingly.

(Correct. CCS 3.2 states that you must ensure that the service you provide is competent and delivered in a timely manner. If you do not agree the work allocation with your supervisor there is a risk that you will not be providing a timely service on the matter that is coming to trial. There is also the possibility that you are too busy to act on the new matter, but another lawyer in your team might be able to do so)

92
Q

A client asks you to act for him and his wife on the re-mortgage of the family home. He explains that he is re-mortgaging the house as security for a loan to finance the expansion of a company he recently started. He is the sole shareholder of the company. He explains that he will be giving the instructions as his wife is overseas visiting family.

Which ONE of the following statements BEST describes what you should do?

A) You can act for the husband and the wife. As the wife is overseas it makes sense for you to act on the husband’s instructions, but you should confirm she is happy for you to act on her behalf when she returns.

B) You can act for the husband and the wife, but you will need authority from the wife to take instructions from her husband. You should talk to the wife to satisfy yourself that she is happy to proceed and get her to confirm her agreement in writing. You will have a continuing obligation to act in the best interests of both the husband and the wife, so if you become concerned that the re-mortgage is not in the wife’s best interests you will need to stop acting for her.

C) You should recommend that the wife obtains her own independent advice on the re-mortgage because there is a risk that you would not be acting in the wife’s best interests if you acted for both the husband and the wife.

D) You can act for the husband and the wife, but you will need authority from the wife to take instructions from her husband. You should talk to the wife to satisfy yourself that she is happy to proceed and get her to confirm her agreement in writing. Once you have this authority you will be able to act for both the husband and the wife.

A

C) You should recommend that the wife obtains her own independent advice on the re-mortgage because there is a risk that you would not be acting in the wife’s best interests if you acted for both the husband and the wife

(Correct. There is a risk that you would not be acting in the wife’s best interests if you acted for both the husband and the wife here. If the husband defaults on the loan, the wife could lose her home and she does not appear to be benefitting from the loan as it is to expand the husband’s business. You should recommend that the wife obtains her own independent advice)

93
Q

Which ONE of the following is the correct definition of an own interest conflict as set out in the SRA Glossary?

A) Any situation where your duty to act in the best interests of any client in relation to a matter conflicts with the interests of an existing client in relation to that matter

B) Any situation where your duty to act in the best interests of any client in relation to a matter conflicts, or there is a significant risk that it may conflict, with your own interests in relation to that or a related matter

C) Any situation where your duty to act in the best interests of any client conflicts with your own interests or the interests of a member of your family in relation to that matter.

A

B) Any situation where your duty to act in the best interests of any client in relation to a matter conflicts, or there is a significant risk that it may conflict, with your own interests in relation to that or a related matter

94
Q

Which ONE of the following options correctly sets out the circumstances in which you can act where there is a conflict of interest?

A) There are no circumstances in which you can act where there is a conflict of interest.

B) The clients have a substantially common interest in relation to the matter or the aspect of it, as appropriate; or the clients are competing for the same objective, and the conditions set out in CCS 6.2(i)-(iii) are met.

C) The clients have a substantially common interest in relation to the matter or the aspect of it, as appropriate; or the clients are competing for the same objective.

A

B) The clients have a substantially common interest in relation to the matter or the aspect of it, as appropriate; or the clients are competing for the same objective, and the conditions set out in CCS 6.2(i)-(iii) are met

(This is the correct position as set out in CCS 6.2)

95
Q

Which ONE of the following options correctly defines a conflict of interest?

A) A situation where you believe there is a significant risk that you will not be able to act in the best interests of your client because you are already acting for a client with a conflicting interest.

B) A situation where you cannot comply with your duty to uphold the rule of law because the interests of two or more clients in the same matter conflict.

C) A situation where your separate duties to act in the best interests of two or more clients in the same or a related matter conflict.

A

C) A situation where your separate duties to act in the best interests of two or more clients in the same or a related matter conflict.

96
Q

Which ONE of the following circumstances is likely to give rise to an ‘own interest conflict’ as defined in the SRA Glossary?

A) Acting for a client on a dispute against your spouse.

B) A client asks you to represent her in relation to a dispute against ABC Limited. You were instructed to act for ABC on the same dispute yesterday.

C) A client asks you to act on the purchase of a company (‘target’). A company search reveals that the target is a wholly owned subsidiary of an existing client of your firm.

A

A) Acting for a client on a dispute against your spouse

(Correct. This is an example of an own interest conflict. As the client has a dispute against your spouse, there is a risk that the interests of the client could conflict with your interest. There is a risk you may not wish the client’s claim to be successful)

97
Q

Which ONE of the following circumstances could give rise to an ‘own interest conflict’ as defined in the SRA Glossary?

A) You are a legal apprentice in the legal department at a large listed company. A solicitor from the law firm your company instructed on a recent transaction invites you to a day at the firm’s box at Wimbledon.

B) A new client asks you to represent her in relation to a dispute with her employer. You discover that your firm acted for the employer 6 months ago on an international restructuring project.

C) A client asks you to represent her in relation to a dispute with her employer. When you read the email from your client, you realise that your mother owns 50% of the shares in the company which employs her.

A

C) A client asks you to represent her in relation to a dispute with her employer. When you read the email from your client, you realise that your mother owns 50% of the shares in the company which employs her.

(Correct. This is an example of an own interest conflict. As your mother owns 50% of the shares in the company employing the client there is a risk that the interests of the client could conflict with your interest. There is a risk you may not wish the client’s claim to be successful)

98
Q

Which one of the following circumstances could give rise to a ‘conflict of interest’ as defined in the SRA Glossary?

A) A new client wants to instruct your firm on a software licence with an international software provider. Your conflict search reveals that your firm’s property department is currently acting for the software provider on its lease of office space.

B) A tenant wants you to advise them on a dispute with their landlord. The tenant does not realise that you and your partner own the flat as the tenancy has been handled by a property agent.

C) A new client has asked your firm to act for them on the purchase of a company from a large international company. Your firm has acted for the international company on a number of acquisitions and disposals in the past, although your firm has not been instructed to act for them on this disposal.

A

C) A new client has asked your firm to act for them on the purchase of a company from a large international company. Your firm has acted for the international company on a number of acquisitions and disposals in the past, although your firm has not been instructed to act for them on this disposal.

(Correct. Conflict of interest is defined in the SRA Glossary as ‘a situation where your separate duties to act in the best interests of two or more clients in the same or a related matter conflict’. You have two clients here, the new client and the international company. Although you have not been instructed by the international company on the same matter, your firm has acted for them on disposals in the past, and this could be a situation where your separate duties to act in the best interests of both clients conflict)

99
Q

Which one of the following statements sets out the CORRECT definition of ‘competing for the same objective’?

A) Any situation in which two or more clients are competing for an “objective” which, if attained by one client, will make that “objective” unattainable to the other client or clients, and “objective” means an asset, contract or business opportunity which two or more clients are seeking to acquire or recover through a liquidation (or some other form of insolvency process) or by means of an auction or tender process of a bid or offer, but not a public takeover.

B) A situation where there is a clear common purpose between the clients and a strong consensus on how it is to be achieved.

C) Any situation in which two or more clients are competing for an “objective” which can only be attained by one of the clients, and “objective” means an asset which two or more clients are seeking to acquire by means of an auction or tender process.

A

A) Any situation in which two or more clients are competing for an “objective” which, if attained by one client, will make that “objective” unattainable to the other client or clients, and “objective” means an asset, contract or business opportunity which two or more clients are seeking to acquire or recover through a liquidation (or some other form of insolvency process) or by means of an auction or tender process of a bid or offer, but not a public takeover.

(Correct. You have correctly identified the definition of competing for the same objective. The other options were the definition of substantially common interest, and an incorrect definition of competing for the same objective.)

100
Q

In which one of the following circumstances might it be possible for a solicitor to act for both the parties stated in bold on the grounds that they have a ‘substantially common interest’ as defined in the SRA Glossary?

A) Each of the partners in Waterloo & Co – a partnership which is being dissolved and selling all of its ass assets to an unassociated third party, Nelsons of Avon Ltd.

B) Su Sukit Singh and Parul Shah who live together and are buying a property together.

C) Bagshaws Bank who have agreed to lend £1 million to Beaumont Limited to fund the acquisition of a company.

A

B) Su Sukit Singh and Parul Shah who live together and are buying a property together

(Correct. ‘Substantially common interest’ is defined in the SRA Glossary as ‘a situation where there is a clear common purpose between the clients and a strong consensus on how it is to be achieved.’ Here the clear common purpose for Sukit and Parul is buying the house. It is likely that they have a strong consensus on how to achieve that purpose, as they will both want to purchase good title to the house without any problems)

101
Q

A company is selling its subsidiary (target) by way of auction. Your firm has been approached by two separate bidders for the Target. Both bidders would like the same partner and associate solicitor to act for them on their bids.

Which of the following statements best states the correct position?

A) As both bidders have a substantially common interest in purchasing the Target, the partner and associate solicitor can act for both bidders.

B) As both bidders are competing for the same objective (the Target), the firm can act for both bidders provided different teams of lawyers act for each bidder.

C) As both bidders are competing for the same objective (the Target), the partner and associate solicitor can act for both bidders.

D) As there is a significant risk of a conflict of interest arising if you act for both bidders, your firm cannot act for either of them.

A

B) As both bidders are competing for the same objective (the Target), the firm can act for both bidders provided different teams of lawyers act for each bidder

(Correct. There is a conflict of interests here, but this is a case where both bidders are competing for the same objective (the Target). The firm can act for both bidders provided they meet the conditions set out in CCS 6.2. One of the conditions is that you put in place effective safeguards to protect your clients’ confidential information. If the same partner and associate solicitor act for both bidders, the confidential information of each bidder will not be protected. The firm should therefore ensure that different teams of lawyers act for each bidder)

102
Q

Which provision of the SRA Codes requires law firms to carry out client conflict checks?

A) SRA Code of Conduct for Solicitors, Registered European Lawyers (RELs) and Registered Foreign Lawyers (RFLs) 6.2

B) SRA Code of Conduct for Solicitors, Registered European Lawyers (RELs) and Registered Foreign Lawyers (RFLs) 6.1

C) SRA Code of Conduct for Firms (‘CCF’) 2.1

A

C) SRA Code of Conduct for Firms (‘CCF’) 2.1

(Correct. CCF 2.1 states that you must ‘have effective governance structures, arrangements, systems and controls in place to ensure:
You comply with all the SRA’s regulatory arrangements, as well as with other regulatory and legislative requirements which apply to you’.
CCF 2.1 means that law firms need to have systems in place to ensure that there are no conflicts in place at the outset of the instruction)

103
Q

John Smith has approached your firm to act for him on the sale of all his shares in John Smith Films Ltd (‘John Smith Films’) to Galaxy Inc (‘Galaxy’), a well-known film studio. Mars Inc (‘Mars’) has also been reported in the press as making an offer to John Smith for his shares in John Smith Films.

Which ONE of the following options correctly states the names against which you should carry out a conflict check?

A) John Smith, John Smith Films, Galaxy and Mars.

B) John Smith and Galaxy

C) John Smith and John Smith Films

D) John Smith, John Smith Films, and Galaxy.

A

D) John Smith, John Smith Films, and Galaxy.

(Correct. You should carry out a conflict check against your client, John Smith, the target, John Smith Films and the counter party, Galaxy. There is no need to carry out a search against Mars because at this stage it is not involved in the transaction, there have just been reports in the press)

104
Q

Outdoor Pursuits Limited (‘Outdoor’) is an online retailer of outdoor leisure clothes and an existing client of your firm. Outdoor wishes to acquire Gorspex Limited (‘Gorspex’), an online climbing equipment retailer, (the ‘Acquisition’). Your firm has been instructed to advise Outdoor in connection with the Acquisition.

Which ONE of the following statements is correct?

A) You should check that your firm is not acting for Gorspex in relation to the Acquisition.

B) If anyone working at your firm owned shares in Gorspex, your firm would not be able to act on the Acquisition because there would be an ‘own interest conflict’.

C) Your firm might be able to act for Outdoor and Gorspex on the Acquisition as they will be competing for the same objective.

A

A) You should check that your firm is not acting for Gorspex in relation to the Acquisition

(Correct. If your firm was acting for Gorspex, there would be a conflict of interest and your firm would not be able to act for Outdoor.
Outdoor and Gorspex are not competing for the same objective here. The ‘objective’ Outdoor is ‘competing for’ is Gorspex, the target, so Gorspex cannot be competing for itself. Remember that the competing for the same objective exception applies mostly to tenders or auctions, not direct arm’s length sales.
‘Own interest conflict’ is defined in the SRA Glossary as ‘any situation where your duty to act in the best interests of any client in relation to a matter conflicts or there is a significant risk that it may conflict with your own interests in relation to that or a related matter.’ If an individual solicitor owned shares in Gorspex, that solicitor would not be able to act on the acquisition due to their own interest conflict. But one solicitor in the firm owning shares in Gorspex would not prevent another solicitor in that firm from acting for Outdoor on the acquisition)

105
Q

Which option below completes the following sentence correctly?

‘CCS 3.2 requires you to…’

A) consider and take account of your client’s attributes, needs and circumstances.

B) send your client a client care letter within one week of receiving new instructions from them.

C) charge your client a fair and reasonable price for the service.

D) ensure that the service you provide to clients is competent and delivered in a timely manner.

A

D) ensure that the service you provide to clients is competent and delivered in a timely manner.

(Correct. This is an important standard from a client care perspective. You must be competent to give the advice your client requests and you must give it in a timely fashion. Many complaints arise from lawyers being too busy to give the client the level of service they are paying for)

106
Q

‘Sending a client care letter is an SRA requirement under CCS 3.1.’

Which ONE of the following statements is CORRECT?

A) The statement is false because the requirement is under CCS 3.3

B) The statement is false because there is no SRA requirement to send a client care letter.

C) The statement is true.

A

B) The statement is false because there is no SRA requirement to send a client care letter

(Correct. Sending a client care letter is not a mandatory requirement under CCS. However, as the SRA requires that certain information is provided to clients in writing, most firms choose to send client care letters to their clients as a way of demonstrating that they have met the SRA’s requirements)

107
Q

Which ONE of the following would NOT be a permitted reason to terminate your retainer with a client?

A) Terminating your retainer to act on the divorce of a current client to give you time to concentrate on the divorce of a celebrity client who has just instructed you.

B) You are acting for a client on the sale of a company (‘Target’) and you discover that the client is selling the Target to a company in which your brother is a majority shareholder.

C) A two-partner accountancy firm has instructed you to act on the sale of their practice. However, you have not been able to obtain clear instructions from the partners because they disagree on the terms of the sale.

D) The client you are defending in a criminal trial has admitted that they lied on oath in their evidence and they are not willing to change their evidence.

A

A) Terminating your retainer to act on the divorce of a current client to give you time to concentrate on the divorce of a celebrity client who has just instructed you.

(Correct. You would be in breach of Principle 7 (act in the best interests of each client) and Principle 2 (act in a way that upholds public trust and confidence in the solicitors’ profession) amongst others if you did this. You would also be in breach of CCS 1.1)

108
Q

Which ONE of the following statements is INCORRECT?

A) Law firms must ensure that clients are informed in writing at the time of engagement about how a complaint can be made and to whom.

B) Law firms must ensure that clients are informed in writing at the time of engagement about their right to complain about the firm’s service and their charges.

C) Clients do not need to be informed of a law firm’s complaints procedure until you become aware that an issue has arisen which you reasonably believe may lead to the client wanting to make a complaint.

D) Law firms must ensure that clients are informed in writing at the time of engagement about any right they have to make a complaint to the Legal Ombudsman and when such a complaint can be made.

A

C) Clients do not need to be informed of a law firm’s complaints procedure until you become aware that an issue has arisen which you reasonably believe may lead to the client wanting to make a complaint.

(Correct answer. This statement is false. CCS 8.3 requires legal professionals (and this requirement applies to firms under CCF 7.1) to inform their clients in writing at the time of engagement about their right to complain, how a complaint can be made and their right, if any, to make a complaint to the Legal Ombudsman)

109
Q

Which ONE of the following statements is CORRECT?

A) If a law firm does not resolve a complaint from a client within 6 weeks following the making of the complaint, solicitors must ensure that the client is informed, in writing, of any right they have to complain to the Legal Ombudsman and the time frame for doing so.

B) If a law firm does not resolve a complaint from a client within 8 weeks following the making of the complaint, solicitors must ensure that the client is informed, in writing, of any right they have to complain to the Legal Ombudsman and the time frame for doing so.

C) If a law firm does not resolve a complaint from a client within 8 weeks following the event giving rise to the complaint, solicitors must ensure that the client is informed, in writing, of any right they have to complain to the Legal Ombudsman and the time frame for doing so

D) If a law firm does not resolve a complaint from a client within 8 weeks following the event giving rise to the complaint, solicitors must ensure that the client is informed, in writing, of any right they have to complain to the SRA and the time frame for doing so.

A

B) If a law firm does not resolve a complaint from a client within 8 weeks following the making of the complaint, solicitors must ensure that the client is informed, in writing, of any right they have to complain to the Legal Ombudsman and the time frame for doing so.

(Correct. See CCS 8.4 (which applies to firms by virtue of CCF 7.1))

110
Q

Which ONE of the following statements is CORRECT?

A) CCS requires legal professionals to inform their clients in writing at the time of engagement about how their matter will be priced and the likely overall cost of the matter.

B) Law firms may charge clients for the reasonable costs incurred by a firm in dealing with a complaint if the complaint is resolved in favour of the firm.

C) CCS requires legal professionals to inform their clients in writing of who will be carrying out the work for the client, and how the firm will charge for the work.

D) CCS requires legal professionals to ensure that clients receive the best possible information about how their matter will be priced and, both at the time of engagement and when appropriate as their matter progresses, about the likely overall cost of the matter and any costs incurred.

A

D) CCS requires legal professionals to ensure that clients receive the best possible information about how their matter will be priced and, both at the time of engagement and when appropriate as their matter progresses, about the likely overall cost of the matter and any costs incurred.

(Correct. See CCS 8.7. Note that CCS does not require this information to be provided in writing, but it is good practice to put this in writing)

111
Q

Which of the following should be included in a client care letter as a matter of good practice?

i. A clear explanation of how the firm will charge for the work.

ii. The names of the lawyers who will be working on the matter.

iii. A description of the work the firm will carry out.

iv. The responsibilities of the firm and the client.

v. An explanation of the client’s right to complain to the Law Society if it is not satisfied with the firm’s service.

Please choose ONE of the following options:

A) (i), (ii), (iii) and (iv)

B) (i) only

C) (iv) and (v)

D) (i), (ii) and (iii)

A

A) (i), (ii), (iii) and (iv)

(Correct. (i) is good practice as well as helping a firm comply with CCS 8.7. (ii), (iii) and (iv) are good practice and recommended in the SRA’s guidance on client care letters. (v) is not correct, a client would not complain to the Law Society. Clients such as individuals who have a right to complain to the Legal Ombudsman should be informed of this in writing under CCS 8.3)

112
Q

Which ONE of the following statements is CORRECT?

A) A client care letter should include all the generic terms and conditions between a solicitor and the client, but not information about costs as this should be agreed separately.

B) A client care letter should set out what is going to happen and what the client needs to do.

C) A client care letter should be written in a small font size so that all the terms can be included on one page.

A

B) A client care letter should set out what is going to happen and what the client needs to do.

(Correct. These are two recommendations from the SRA guidance on client care letters)

113
Q

Which ONE of the following statements is CORRECT?

A) A law firm must send a client care letter to a client at the beginning of every new instruction.

B) A law firm will have met the SRA’s requirements on client care if it sends a client care letter to its client at the beginning of every matter.

C) A client care letter must set out all the firm’s terms and conditions in full.

D) One benefit of a client care letter that is clear, easy to understand and contains the right information is that it can reduce potential complaints by clearly setting out what a client can expect, particularly in relation to costs. ​

A

D) One benefit of a client care letter that is clear, easy to understand and contains the right information is that it can reduce potential complaints by clearly setting out what a client can expect, particularly in relation to costs. ​

(Correct. This is one benefit of a clearly written client care letter as described in the SRA’s Guidance on Client Care letters)

114
Q

You represent a man suspected of the offence of battery and is charged with common assault. He is currently out of work and so receives welfare benefits.

Which of the following best explains whether you client is eligible for public funding?

A) He will receive public funding because the offence of battery is an imprisonable offence.

B) He will pass the means test if he can show that his income and capital are below a certain limit.

C) He will automatically qualify for public funding because he is in receipt of welfare benefits.

D) He will automatically pass the means test because he is in receipt of welfare benefits but he must also pass the merits test to be eligible.

E) He will have to pass both the means test and the merits test in order to be granted public funding.

A

D) He will automatically pass the means test because he is in receipt of welfare benefits but he must also pass the merits test to be eligible

(He is passported through the means test because of his receipt of welfare benefits, but he will still have to show that it is in the interests of justice for him to receive public funding for his defence.
The other answers are plausible but incorrect:
- He does not need to pass the means test – he automatically passes it
- He won’t automatically qualify for public funding because he still has to pass the merits test
- Whether the offence is imprisonable or not is not a deciding factor)

115
Q

Your client has been charged with burglary. She is in receipt of benefits and therefore is passported through the means test. You are assisting your client in completing the CRM 14 form.

She informs you that she has numerous convictions for burglary, including one last year when she was given a suspended sentence. She denies the offence and says she has an alibi. However, your client is worried about what the arresting officer will say. The arresting officer said she had admitted the offence on arrest, when she did not say anything. She’s also worried about her daughter who is only 12 and will be left on her own if she is sent to prison.

Which of these is not a matter to include in the CRM 14 form?

A) The proceedings will involve the cross examination of a police officer

B) It is in the interests of her daughter that she is represented

C) She is likely to lose her liberty

D) An alibi witness will need to be traced and interviewed

E) She has been given a sentence that is suspended which will be activated if she is convicted of the current offence

A

B) It is in the interests of her daughter that she is represented

(For the purposes of the merits/ interests of justice test, the interests of another person does not include family members impacted by a sentence.
The other answers while plausible are not correct. All could be included in the relevant sections of the CRM 14 form)

116
Q

Your client is a university professor who lives alone and earns £90,000. He is suspected of the offence of battery on a student and is charged with common assault. He is due to appear before the magistrates’ court He has no previous offences and intends to plead not guilty.

Is your client likely to obtain a representation order that grants public funding for his defence?

A) He will obtain a representation order because it is likely he will lose his livelihood

B) He will not obtain a representation order as will fail the means test

C) He will obtain a representation order because it is likely that he will suffer serious damage to his reputation

D) He will not obtain a representation order because it is not in the interests of justice that he be represented

E) He will not obtain a representation order because he is not in receipt of benefits

A

B) He will not obtain a representation order as will fail the means test

(In light of the university professor’s salary and the size of his household (he lives alone) he will be ineligible for public funding because his weighted gross annual income will be above the upper threshold i.e. £22, 325.
The other answers while plausible are incorrect:
· While he will suffer serious damage to his reputation, might lose his livelihood and therefore it might be in the interests of justice that he be represented, he will not pass the means test.
· While he is not in receipt of benefits that does not mean he will automatically fail the means test)

117
Q

Which ONE of the following statements is CORRECT?

A) Any organisation providing reserved legal activities for the public must be authorised by the Law Society unless they are exempt.

B) All organisations providing legal activities for the public must be authorised by the SRA.

C) Any organisation providing reserved legal activities for the public must be authorised by the SRA unless they are exempt.

A

C) Any organisation providing reserved legal activities for the public must be authorised by the SRA unless they are exempt.

(Correct. The SRA Guidance note, “Firm Authorisation” , contains guidance on when organisations need to be authorised by the SRA, including if they provide reserved legal activities)

118
Q

Which one of the following statements is CORRECT?

A) Reserved legal activities are defined in section 12 Legal Services Act 2007 and are: (i) rights of audience: the right to appear before and address a court, (ii) conduct of litigation, (iii) acting for a client on the acquisition of a company, (iv) Reserved instrument activities which includes preparing instruments relating to land, and (v) certain probate activities.

B) Reserved legal activities are defined in section 12 Legal Services Act 2007 and are: (i) rights of audience: the right to appear before and address a court, (ii) conduct of litigation, (iii) Reserved instrument activities which includes preparing instruments relating to land, (iv) certain probate activities; and (v) administration of oaths.

C) Reserved legal activities are defined in section 12 Legal Services Act 2007 and are: (i) rights of audience: the right to appear before and address a court, (ii) Reserved instrument activities which includes preparing instruments relating to land; and (iii) administration of oaths.

A

B) Reserved legal activities are defined in section 12 Legal Services Act 2007 and are: (i) rights of audience: the right to appear before and address a court, (ii) conduct of litigation, (iii) Reserved instrument activities which includes preparing instruments relating to land, (iv) certain probate activities; and (v) administration of oaths

(Correct. Reserved legal activities are defined in section 12 LSA and are: (i) rights of audience: the right to appear before and address a court, (ii) conduct of litigation, (iii) Reserved instrument activities which includes preparing instruments relating to land, (iv) certain probate activities; and (v) administration of oaths. The term does not include acting for a client on the acquisition of a company)

119
Q

Which ONE of the following statements is CORRECT?

A) ‘Sole practice’ means a sole principal. A sole practitioner can employ several qualified solicitors, as long as those solicitors are not also principals (e.g. not partners or co-owners) of the practice.

B) A sole practice is a practice operated by a single solicitor. A sole practitioner can employ other members of staff provided they are not qualified solicitors.

C) A sole practice is a practice operated by a single solicitor. A sole practitioner can employ a maximum of three other members of staff provided they are not qualified solicitors.

A

A) ‘Sole practice’ means a sole principal. A sole practitioner can employ several qualified solicitors, as long as those solicitors are not also principals (e.g. not partners or co-owners) of the practice.

(Correct. ‘Sole practice’ means a sole principal. A sole practitioner can employ several qualified solicitors, as long as those solicitors are not also principals (e.g. not partners or co-owners) of the practice)

120
Q

The SRA publishes a risk outlook each year. Which ONE of the following is the risk the SRA put at the top of its priority list for the year 2020-21?

A) Keeping client money safe

B) Money Laundering

C) Information and cyber security

D) Increasing diversity in the legal profession.

A

B) Money Laundering

(Correct. The SRA put issues with money laundering at the tops of its priority risk in the Risk Outlook for 2020-21)

121
Q

Which ONE of the following statements is CORRECT?

A) It is the responsibility of individual solicitors to keep their knowledge up to date, but it is the sole responsibility of the partners in law firms to ensure that the individuals they manage provide a competent service to their clients.

B) It is the responsibility of supervisors to ensure that the individuals they manage are competent to carry out their role and keep their professional knowledge and skills up to date. There is no corresponding obligation on individual solicitors to keep their knowledge up to date.

C) It is the responsibility of supervisors to ensure that the individuals they manage are competent to carry out their role and keep their professional knowledge and skills up to date. Individual solicitors also have a responsibility under CCS to maintain their competence and keep their knowledge up to date.

A

C) It is the responsibility of supervisors to ensure that the individuals they manage are competent to carry out their role and keep their professional knowledge and skills up to date. Individual solicitors also have a responsibility under CCS to maintain their competence and keep their knowledge up to date.

(Correct. CCS 3.3 requires individual lawyers to maintain their competence to carry out their role and keep their professional knowledge and skills up to date. CCS 3.6 requires legal professionals to ensure that the individuals they manage are competent to carry out their role and keep their professional knowledge and skills up to date. So the responsibility falls on both individual lawyers and supervisors)

122
Q

Which ONE of the following statements is CORRECT?

A) All people supervising individuals working on client files in a law firm must be legally qualified.

B) Only solicitors with a minimum of five years post qualification experience may supervise individuals working on client files.

C) Although most persons supervising individuals working on client files will have a legal qualification, such persons do not necessarily have to be legally qualified.

A

C) Although most persons supervising individuals working on client files will have a legal qualification, such persons do not necessarily have to be legally qualified.

(Correct. As long as they have suitable experience, knowledge and competence to deal with any issue which may arise and such persons have clear guidance as to when and to whom issues outside their competence or authority should be referred ‘upwards’, people who are not legally qualified are permitted to supervise client matters)

123
Q

Which one of the following statements is CORRECT?

A) The COFA has responsibility:
to ensure the firm complies with all the terms and conditions of authorisation by the SRA;
to ensure the firm complies with its statutory obligations;
to record any failures to comply with the firm’s authorisation or statutory obligations and make records available to the SRA;
to report any material failure to the SRA as soon as is practicable.

B) The COLP and COFA are jointly responsible:
to ensure the firm complies with all the terms and conditions of authorisation by the SRA;
to ensure the firm complies with its statutory obligations;
to record any failures to comply with the firm’s authorisation or statutory obligations and make records available to the SRA;
to report any material failure to the SRA as soon as is practicable

C) The COLP has responsibility:
to ensure the firm complies with all the terms and conditions of authorisation by the SRA;
to ensure the firm complies with its statutory obligations;
to record any failures to comply with the firm’s authorisation or statutory obligations and make records available to the SRA;
to report any material failure to the SRA as soon as is practicable.

A

C) The COLP has responsibility:
to ensure the firm complies with all the terms and conditions of authorisation by the SRA;
to ensure the firm complies with its statutory obligations;
to record any failures to comply with the firm’s authorisation or statutory obligations and make records available to the SRA;
to report any material failure to the SRA as soon as is practicable.

(Correct. The COLP’s role is to ensure appropriate systems are in place to minimise the risk of non-compliance with the Codes. Much of the COLP’s day to day responsibilities will be spent trying to ensure that the firm, its staff and managers are meeting the standards in CCF and CCS)

124
Q

Which one of the following statements is CORRECT?

A) The COLP and the COFA are the people in a law firm who bear ultimate responsibility to the SRA for ensuring the firm complies with the SRA’s regulatory requirements under the SRA Code of Conduct for Firms.

B) All the partners and solicitors in a firm are jointly responsible for the firm’s compliance with the SRA Code of Conduct for Firms.

C) The managers of a firm have joint and several responsibility for the firm’s compliance with the SRA Code of Conduct for Firms.

A

C) The managers of a firm have joint and several responsibility for the firm’s compliance with the SRA Code of Conduct for Firms.

(Correct. Compliance with the SRA’s requirements is ultimately the responsibility of the owners and managers of the Firm. Under CCF 8.1 managers have responsibility for compliance by the Firm with CCF. This responsibility will be joint and several with other managers of the firm. Nevertheless, COLPs and COFAs clearly have a pivotal function in their organisation for ensuring that the right systems, procedures and checks are in place to minimise the risk of non-compliance)

125
Q

Which ONE of the following statements is CORRECT?

A) The Compliance Officer for Legal Practice (‘COLP’) and the Compliance Officer for Finance and Administration (‘COFA’) must both be individuals who are authorised to carry on reserved legal activities by an approved regulator.

B) The Compliance Officer for Legal Practice (‘COLP’) and the Compliance Officer for Finance and Administration (‘COFA’) must both be legally qualified.

C) The Compliance Officer for Legal Practice (‘COLP’) must be an individual who is authorised to carry on reserved legal activities by an approved regulator. This is not a requirement for the Compliance Officer for Finance and Administration (‘COFA’).

A

C) The Compliance Officer for Legal Practice (‘COLP’) must be an individual who is authorised to carry on reserved legal activities by an approved regulator. This is not a requirement for the Compliance Officer for Finance and Administration (‘COFA’).

(Correct. This requirement is true for a COLP, but not a COFA. A COFA’s role is to ensure the firm complies with the SRA Accounts Rules and as such the COFA does not need to be a lawyer)

126
Q

Which ONE of the following correctly states two of the largest liabilities of a law firm?

A) Fees paid by clients and marketing

B) Rent and salaries

C) Training events for staff and client entertainment

A

B) Rent and salaries

(Correct. Two of the main liabilities of a law firm are rent and salaries. Many firms are based in central city locations to be close to the businesses which they represent. The downside to this is, of course, the higher rent. Equally, while more staff mean that more work can be taken on, the salaries of those staff are considerable.
Fees paid by clients are the main source of income for a law firm, not outgoings. Marketing expenses is an outgoing but should not be one of the largest two liabilities of a law firm. Training events for staff should not be one of the largest liabilities of a law firm although it is an outgoing. Expenditure on client entertainment is also an outgoing, but again should not be one of the largest outgoings of a law firm)

127
Q

Which one of the following statements is CORRECT?

A) Hourly charging is the traditional method of billing a client. However, it is coming under increasing pressure as clients seek to control the extent of their legal costs, and other methods of billing are therefore becoming more common.

B) Hourly charging is the most common method of billing clients used by law firms.

C) Hourly charging is the method of billing most favoured by clients as it enables clients to see how much time has been spent on a matter by all the lawyers working on it.

A

A) Hourly charging is the traditional method of billing a client. However, it is coming under increasing pressure as clients seek to control the extent of their legal costs, and other methods of billing are therefore becoming more common

(Correct. The problem with hourly charging from the client’s perspective is that there is no limit on the amount the firm charges the client for the work. If the lawyers take 100 hours to complete the project, the firm could charge for the full amount, assuming the partner responsible for the matter considers that to be a reasonable amount for the work. Clients are therefore increasingly looking to agree other billing methods, such as fixed fees or a cap on legal fees to help them manage the amount they spend on external lawyers)

128
Q

Which one of the following statements CORRECTLY defines a fixed fee agreement?

A) In a fixed fee agreement, the law firm will record (and bill) their fees in the normal way, but the fees will not exceed a pre-agreed amount for the particular transaction.
If the recorded time on a file reaches the fixed amount, the legal team may be asked to stop work on the file while the partner attempts to renegotiate the fixed amount.
Otherwise, work will continue as normal past the fixed amount, and any fees incurred above the amount will have to be written off.

B) In a fixed fee agreement, the law firm will invoice a client for a fixed amount, regardless of the amount of time spent by fee earners on the transaction. The fixed amount will be fixed at the agreed amount whether the hours worked by the lawyers on an hourly charging basis work out as more or less than the fixed fee. Fixed fee agreements are common for conveyancing or wills eg drafting a will for a set fee of £95 plus VAT

C) Fixed fee agreements are common in litigation and mediation cases. Under fixed fee agreements, the lawyers do not recover any fees if they lose the case. If they win, they may recover their fees out of the damages awarded to the client. The maximum amount the lawyer can recover is capped, eg at 25% for personal injury cases. There are rules in place as to when and how fixed fee agreements can be used and enforced.

A

B) In a fixed fee agreement, the law firm will invoice a client for a fixed amount, regardless of the amount of time spent by fee earners on the transaction. The fixed amount will be fixed at the agreed amount whether the hours worked by the lawyers on an hourly charging basis work out as more or less than the fixed fee. Fixed fee agreements are common for conveyancing or wills eg drafting a will for a set fee of £95 plus VAT

(Correct. In a fixed fee agreement the law firm will charge the client the fixed fee whether the hours worked by the lawyers on the task work out as more or less than the fixed fee. If the hours worked by the lawyers work out as more than the fixed fee the firm will make a loss, if the hours work out as less than the fixed fee, the firm will be able to charge the full amount of the fixed fee. Fixed fees tend to be used for projects the firm carries out regularly where they can confidently predict the number of hours it will take to complete the work.
A capped fee agreement is where the law firm will record (and bill) their fees in the normal way, but the fees will not exceed a pre-agreed amount for the particular transaction. If the recorded time on a file reaches the cap, the legal team may be asked to stop work on the file while the partner attempts to renegotiate the cap. Otherwise, work will continue as normal past the cap, and any fees incurred above the amount will have to be written off.
A conditional fee agreement is common in litigation and mediation cases. The lawyers do not recover any fees if they lose the case. If they win, they may recover their fees out of the damages awarded to the client. The maximum amount the lawyer can recover is capped, eg at 25% for personal injury cases. There are rules in place as to when and how conditional fee agreements can be used and enforced.)

129
Q

Which one of the following statements about partnerships is CORRECT

A) One of the main disadvantages of a traditional partnership is that they are governed by the Partnership Act 1890 which is inflexible and largely out of date.

B) One advantage of a traditional partnerships is that although they are governed by the Partnership Act 1890, in practice most of their arrangements are set out in partnership agreements, which are not a matter of public record.

C) One advantage of a traditional partnership is that the liability of the partners is limited to the amount they have invested in the partnership.

A

B) One advantage of a traditional partnerships is that although they are governed by the Partnership Act 1890, in practice most of their arrangements are set out in partnership agreements, which are not a matter of public record.

(Correct. Although technically partnerships are governed by the Partnership Act 1890, in practice most partnerships agree the arrangements for how they are run in partnership agreements. There are no requirements on partnerships to disclose the contents of their partnership agreements which means that arrangements relating to profit sharing and running the partnership can be kept confidential. One disadvantage of a traditional partnership is that the liability of individual partners is unlimited opening the partners to potentially large exposure of risk)

130
Q

Which ONE of the following statements about Limited Liability Partnerships (‘LLPs’) is CORRECT?

A) An LLP is a separate legal entity, distinct from its members. An LLP can own property in its own name, and employ employees. However one disadvantage of an LLP is that they have to file certain information with Companies House, including financial statements, which can be accessed by the public.

B) An LLP is a separate legal entity, distinct from its members. An LLP can own property in its own name, and employ employees. They are also more tax efficient than incorporated companies.

C) An LLP is a separate legal entity, distinct from its members. An LLP can own property in its own name, and employ employees. They are also recognised in foreign jurisdictions making foreign expansion easier than for a traditional partnership.

A

A) An LLP is a separate legal entity, distinct from its members. An LLP can own property in its own name, and employ employees. However one disadvantage of an LLP is that they have to file certain information with Companies House, including financial statements, which can be accessed by the public

(Correct. An LLP is a separate legal entity, distinct from its members. An LLP can own property in its own name, and employ employees. LLPs are subject to many provisions of the Companies Act 2006, which means that they have to file certain information with Companies House, including financial statements and information about the members (or partners), which can be accessed by the public.
LLPs are generally considered to be less tax efficient than incorporated companies. In addition unlike incorporated companies, they are not recognised in foreign jurisdictions so foreign expansion is more difficult)

131
Q

Which ONE of the following statements is CORRECT?

A) Incorporated companies can raise finance by issuing shares to investors but cannot borrow money from lenders because they are separate legal entities and cannot grant security to lenders.

B) Traditional partnerships can use working capital to finance growth but cannot borrow money or issue shares to investors

C) Incorporated companies can raise finance by issuing shares to investors and can borrow money from lenders.

A

C) Incorporated companies can raise finance by issuing shares to investors and can borrow money from lenders.

(Correct. Incorporated companies are separate legal entities in their own right. They can borrow money from lenders, grant security to lenders, and they can also raise finance by issuing shares to investors.
Traditional partnerships can use working capital to finance growth, and can also use overdrafts or take out loans from lenders. Traditional partnerships cannot issue shares to investors)

132
Q

Which ONE of the following statements is CORRECT?

A) The money laundering regulations apply to corporate solicitors in law firms but not to other areas of practice or to in-house lawyers or other industries.

B) Every lawyer, whether they work in a law firm or in-house, is at risk of exposure to money laundering.

C) The money laundering regulations apply to law firms but not to in-house lawyers or other industries.

A

B) Every lawyer, whether they work in a law firm or in-house, is at risk of exposure to money laundering

(Correct. The Money Laundering, Terrorist Financing and Transfer of Funds (Information on the Payer) Regulations 2017 (‘MLR 2017’) apply to a wide range of people and businesses, not just law firms)

133
Q

Which ONE of the following statements is CORRECT?

A) Money laundering can include receiving small proceeds of minor crimes into your employer’s bank account.

B) There is a risk of a firm being involved in money laundering only when it receives proceeds of crimes in excess of £10,000 into its bank account.

C) There is a risk of a firm being involved in money laundering only when it receives proceeds of crimes in excess of £100,000 into its bank account.

A

A) Money laundering can include receiving small proceeds of minor crimes into your employer’s bank account

(Correct. Receiving the proceeds of crime, however small into your bank account can constitute possessing criminal property and could be a criminal offence)

134
Q

Which option correctly identifies the key pieces of legislation relating to money laundering in the UK?

A) The Financial Services and Markets Act 2000 (‘FSMA’) and the Money Laundering Regulations 2007 (‘MLR’)

B)The Proceeds of Crimes Act 2002 (‘PoCA’) and the Money Laundering Regulations 2007 (‘MLR’)

C) The Proceeds of Crime Act 2002 (‘PoCA’), and the Money Laundering, Terrorist Financing and Transfer of Funds (Information on the Payer) Regulations 2017 (‘MLR’)

A

C) The Proceeds of Crime Act 2002 (‘PoCA’), and the Money Laundering, Terrorist Financing and Transfer of Funds (Information on the Payer) Regulations 2017 (‘MLR’)

(Correct, these are the two key pieces of legislation relating to money laundering in the UK)

135
Q

Which ONE of the following statements is CORRECT?

A) A high risk instruction occurs where you have a suspicious attribute and a suspicious client.

B) A high risk instruction occurs where you have a suspicious attribute and a suspicious activity.

C) A high risk instruction occurs where you have a suspicious amount of money and a suspicious activity.

A

B) A high risk instruction occurs where you have a suspicious attribute and a suspicious activity.

136
Q

In which sections of the Proceeds of Crime Act 2002 (‘PoCA’) are the direct involvement offences contained?

A) Sections 327, 328 and 329 PoCA

B) Sections 327, 330 and 340 PoCA.

C) Sections 329, 330, and 333A PoCA.

A

A) Sections 327, 328 and 329 PoCA

(Correct. Section 327 contains the direct involvement offences are contained in s. 327 ( concealing, disguising, converting or transferring criminal property), 328 (enters arrangements concerning criminal property) and 329 (acquisition, use and possession of criminal property). S. 330 relates to the offence of failure to disclose which is not a direct involvement, s. 333A relates to the offence of tipping off (also not a direct involvement offence) and s. 340 contains definitions including the definition of criminal property)

137
Q

Which ONE of the following options completes the following sentence CORRECTLY:

It is a defence to the offence of concealing, disguising, converting or transferring criminal property to:

A) “report your suspicion to a nominated officer”

B) “report your suspicion to the SRA”

C) “report your suspicion to your manager”

A

A) “report your suspicion to a nominated officer”

(Correct. You have a defence to the offence of concealing, disguising, converting or transferring criminal property under s. 327 PoCA if you made an authorised disclosure. Under s. 338 the disclosure should be made to a constable, customs office or a nominated officer. In practice the most practical thing to do is therefore to report your suspicion to your employer’s nominated officer)

138
Q

In which ONE of the following circumstances are you NOT required need to carry out customer due diligence (‘CDD’) under the the Money Laundering, Terrorist Financing and Transfer of Funds (Information on the Payer) Regulations 2017 (‘MLR’)?

A) When you establish a business relationship

B) When you carry out an occasional transaction.

C) When you submit a proposal to a prospective client in a beauty contest to provide them with legal services.

A

C) When you submit a proposal to a prospective client in a beauty contest to provide them with legal services.

(Correct. Under regulation 27 MLR a relevant person must carry out CDD if they establish a business relationship, carry out an occasional transaction, suspect money laundering, or doubt the veracity of documents or information provided by the client.
When you submit a proposal to a client in a beauty contest, you don’t know whether you will win the work, so it is too early to carry out CDD)

139
Q

Which ONE of the following statements is CORRECT?

A) Technically a solicitor may rely on CDD carried out by another law firm with the consent of that law firm, but in practice this is rare partly because the solicitor will remain liable for any failure to carry out CDD.

B) A solicitor can rely on CDD carried out by another law firm, but not if the first law firm conducted the CDD more than six month’s previously.

C) A solicitor can never rely on CDD carried out by another person.

A

A) Technically a solicitor may rely on CDD carried out by another law firm with the consent of that law firm, but in practice this is rare partly because the solicitor will remain liable for any failure to carry out CDD

(Correct. A solicitor may rely on CDD carried out by another person under regulation 39, but the solicitor will remain liable for any failure to apply CDD so in practice it is rare to rely on the CDD carried out by another person)

140
Q

Which regulation of the MLR sets out the definition of a business relationship?

A) Regulation 5

B) Regulation 4

C) Regulation 3

A

B) Regulation 4

(Correct. Regulation 4 sets out the definition of business relationship. The definition of occasional transaction is set out in regulation 3. The definition of beneficial owner is set out in regulation 5)

141
Q

Which regulation of the MLR sets out the requirements for enhanced CDD?

A) Regulation 12

B) Regulation 37

C) Regulation 33

A

C) Regulation 33

(Correct. Regulation 33 sets out the requirements for CDD. Regulation 37 sets out the requirements for simplified CDD and regulation 12 sets out the definition of independent legal professional)

142
Q

Which ONE of the following statements about the Money Laundering, Terrorist Financing and Transfer of Funds (Information on the Payer) Regulations 2017 (‘MLR’) is CORRECT?

A) The MLR apply to financial institutions, some legal work, auditors, estate agents and casinos.

B) The MLR apply to financial institutions, all legal work and auditors.

C) The MLR apply to all legal work and financial institutions.

A

A) The MLR apply to financial institutions, some legal work, auditors, estate agents and casinos

(Correct. These organisations are listed in Regulation 8(2) MLR. The MLR do not apply to all types of legal work, only those falling within Regulation 8(2))

143
Q

Which ONE of the following statements is CORRECT?

A) The MLR are designed to ensure that organisations that take part in money laundering will be criminally liable.

B) The MLR are designed to ensure that organisations establish procedures to forestall and prevent operations relating to money laundering.

C) The MLR are designed to ensure that organisations report their suspicions of money laundering to the police.

A

B) The MLR are designed to ensure that organisations establish procedures to forestall and prevent operations relating to money laundering.

(Correct. The Proceeds of Crime Act 2002 anticipates the reporting of suspicious transactions involving the proceeds of crime. MLR requires organisations to get themselves in a position to identify potential money laundering and report it as necessary)

144
Q

Which ONE of the following statements is CORRECT?

A) A solicitor advising a company on the sale of a business entity is likely to fall within the definition of independent legal professional and be caught by the MLR.

B) A solicitor who advises their client on a litigation claim for personal injury is likely to fall within the definition of independent legal professional and be caught by the MLR.

C) A solicitor who sets up a company for a client is not likely to fall within the definition of independent legal professional and be caught by the MLR

A

A) A solicitor advising a company on the sale of a business entity is likely to fall within the definition of independent legal professional and be caught by the MLR

(Correct. A solicitor advising a company on the sale of a business entity would fall within regulation 12(1)(a) MLR, “the buying and selling of real property” and would therefore fall within the definition of an independent legal professional.
A solicitor who sets up a company for a client is likely to fall within the definition of independent legal professional under regulation 12(1) (e) “ the creation, operation of management of trusts, companies…” and also “trust or company service provider” under regulation 12(2)(a), “forming companies”, and would therefore be caught by MLR.
A solicitor who advises their client on a personal injury litigation matter would not fall within the definition of independent legal professional and is therefore not likely to be caught by the MLR)

145
Q

for her daughter for a price of £750,000. She is funding 50% of the purchase in cash from the sale of some shares, and 50% by a mortgage with a high street bank. You have not acted for the client before and you expect that this will be a one-off transaction for the client.

Which ONE of the following options correctly states the evidence of identity and verification you should obtain to demonstrate you have carried out the required level of CDD on this client?

A) Assuming you meet your client face to face, producing a valid passport or photocard ID should meet the CDD requirements.

B) A utility bill or bank statement showing the client’s address for the last three years.

C) A copy of the title deeds for the client’s current address or if her accommodation is rented, a copy of the rental agreement.

A

A) Assuming you meet your client face to face, producing a valid passport or photocard ID should meet the CDD requirements

(Correct. This is the evidence recommended in the guidance on anti-money laundering regulations (‘AMLG’) published by the Legal Sector Affinity Group. It is good practice to have either one ‘government document’ which verifies either the name and address or name and date of birth; or a ‘government document’ which verifies the client’s full name and another supporting document which verifies the name and either their address or date of birth)

146
Q

You have been instructed by a company Autolext Corporation (‘Autolext’)on the purchase of 60% of the shares in a data analytics company, Data Inc for £2 million. Data Inc is registered in the Cayman Islands. Autolext is registered in Bermuda and has a majority shareholder who is Iranian. Autolext has asked to you set up a wholly owned subsidiary, Autolext Data Limited which will be the vehicle that purchases the shares in the Data Inc. Autolext will pay cash for the shares.

Which ONE of the following options correctly states the evidence of identity and verification you should obtain to demonstrate you have carried out the required level of CDD on this client?

A) You should identify the name, registered number and address of the registered office of Autolext, and verify it, for example by a search of the relevant company registry in Bermuda. You must also take reasonable measure to determine and verify the law to which the company is subject, its constitution, and the full names of the board of directors (or equivalent management body) and senior persons responsible for its operations. In addition you will need to examine the background and purpose of the transaction, to determine whether the transaction or relationship appear suspicious. This would involve making further enquiries to understand the financial situation of the company, getting additional independent sources to verify the information you obtain and satisfying yourself that the transaction is legitimate.

B) You should identify the name, registered number and address of the registered office of Autolext, and verify it, for example by a search of the relevant company registry in Bermuda. You must also take reasonable measure to determine and verify the law to which the company is subject, its constitution, and the full names of the board of directors (or equivalent management body) and senior persons responsible for its operations.

C) There is a high risk of money laundering here, so you will need to examine the background and purpose of the transaction, to determine whether the transaction or relationship appear suspicious. This would involve making enquiries to understand the background and financial situation of Autolext, getting additional independent sources to verify the information you obtain and satisfying yourself that the transaction is legitimate.

A

A) You should identify the name, registered number and address of the registered office of Autolext, and verify it, for example by a search of the relevant company registry in Bermuda. You must also take reasonable measure to determine and verify the law to which the company is subject, its constitution, and the full names of the board of directors (or equivalent management body) and senior persons responsible for its operations. In addition you will need to examine the background and purpose of the transaction, to determine whether the transaction or relationship appear suspicious. This would involve making further enquiries to understand the financial situation of the company, getting additional independent sources to verify the information you obtain and satisfying yourself that the transaction is legitimate

(Correct. There are a number of factors that suggest that there is a high risk of money laundering in this scenario and you therefore need to carry out enhanced CDD. This means that in addition to carrying out ‘standard CDD’ of identifying and verifying the identity of Autolext, you will need to carry out further enquiries to satisfy yourself that the client and the transaction is legitimate.)

147
Q

Regency Investments PLC is listed on the London Stock Exchange and is an existing client of your firm. It regularly instructs you on the acquisition and sale of properties in its portfolio. You have received a new instruction to act for it on the acquisition of the freehold property for £7 million.

Which ONE of the following options correctly states the evidence of identity and verification you should obtain and CDD you should carry out on this client?

A) As Regency Investment PLC is an existing client, you should already have carried out CDD on it. As a listed company, it is likely that you will have carried out simplified CDD for example by obtaining a copy of the dated page of the website of the relevant stock exchange showing the listing, and verifying it by a search of the company registry. You will need to conduct ongoing monitoring by checking that the identification documents you hold are up to date and checking the transaction is consistent with your firm’s knowledge of the client.

B) You should carry out standard CDD by identifying the name, registered number and address of the registered office of Regency Investment PLC, and verifying it, for example by a search of the company registry. You must also take reasonable measure to determine and verify the law to which the company is subject, its constitution, and the full names of the board of directors (or equivalent management body) and senior persons responsible for its operations.

C) As Regency Investment PLC is a listed public company, you should carry out simplified CDD for example by obtaining copy of the dated page of the website of the relevant stock exchange showing the listing, and verifying it by a search of the company registry.

A

A) As Regency Investment PLC is an existing client, you should already have carried out CDD on it. As a listed company, it is likely that you will have carried out simplified CDD for example by obtaining a copy of the dated page of the website of the relevant stock exchange showing the listing, and verifying it by a search of the company registry. You will need to conduct ongoing monitoring by checking that the identification documents you hold are up to date and checking the transaction is consistent with your firm’s knowledge of the client

(Correct. This is an existing client whom the firm acts for on a regular basis and a listed public company. It is likely the firm will already have carried out simplified CDD on the client. The firm will need to carry out ongoing monitoring, by checking that the transaction is consistent with the firm’s knowledge of the client and checking at regular intervals that the identification documents it holds for the client are up to date)

148
Q

Which ONE of the following options best states why financial services are regulated in the UK?

A) There are many different investment products available on the market and individuals increasingly look to professional advisers to help them decide which investment to make. The provision of financial services is regulated in the UK to ensure that consumers are adequately protected from negligent financial advice.

B) There are many complex investment products available on the market. The provision of financial services is regulated in the UK to ensure that you can only advise on financial products if you have a maths degree.

C) There are many different investment products available on the market and individuals increasingly look to professional advisers to help them decide which investment to make. The provision of financial services is regulated in the UK to ensure that consumers only invest in safe financial products.

D) There are many different investment products available on the market and individuals increasingly look to professional advisers to help them decide which investment to make. The provision of financial services is regulated in the UK to ensure that the banks that offer the products do not collapse.

A

A) There are many different investment products available on the market and individuals increasingly look to professional advisers to help them decide which investment to make. The provision of financial services is regulated in the UK to ensure that consumers are adequately protected from negligent financial advice

(Correct. Individuals need to make decisions on difficult issues including the level of risk they should take and the return they seek when making an investment. Financial advisors in the UK must be authorised to ensure that they do not give negligent advice to their clients)

149
Q

Which ONE of the following statements is CORRECT?

A) There are two regulators of financial services in the UK, the Bank of England and the Financial Services Authority.

B) There are two regulators of financial services in the UK, the Prudential Regulation Authority and the Financial Conduct Authority.

C) There are two regulators of financial services in the UK, the Bank of England and the Financial Conduct Authority.

A

B) There are two regulators of financial services in the UK, the Prudential Regulation Authority and the Financial Conduct Authority

(Correct. The two regulators of financial services in the UK are the Prudential Regulation Authority and the Financial Conduct Authority. The Prudential Regulation Authority is currently a subsidiary of the Bank of England)

150
Q

Which ONE of the following statements is CORRECT?

A) If you were to carry out a regulated activity, for example advising on the merits of an investment, without being authorised by the Financial Conduct Authority to do so, or exempt, this would be a serious matter of misconduct which would be determined by the Solicitors Disciplinary Tribunal.

B) It is a criminal offence to carry on a regulated activity, for example advising on the merits of an investment, unless you are authorised by the Financial Conduct Authority to do so, or exempt.

C) It is a civil offence to carry on a regulated activity, for example advising on the merits of an investment, unless you are authorised by the Financial Conduct Authority to do so, or exempt.

A

B) It is a criminal offence to carry on a regulated activity, for example advising on the merits of an investment, unless you are authorised by the Financial Conduct Authority to do so, or exempt

(Correct. See section 19 Financial Services and Markets Act 2000 (FSMA))

151
Q

Which ONE of the following would be MOST HELPFUL for you in navigating the FSMA decision tree?

A) SRA Code of Conduct for Solicitors, RELs, and RFLs 2019

B) Financial Services Act 2012

C) Financial Services and Markets Act 2000 (Regulated Activities) Order 2001

A

C) Financial Services and Markets Act 2000 (Regulated Activities) Order 2001

(Correct. The Financial Services and Markets Act 2000 (Regulated Activities) Order 2001 (‘RAO’) contains the majority of the detail you will need to navigate the FSMA decision tree. Part III lists the specified investments, and Part II lists the specified activities. In addition, each chapter within Part II RAO lists the specified activity and then the exclusions that apply to that specified activity. This is very helpful in helping you navigate questions 1, 2 and 3 of the FSMA decision tree)

152
Q

Which section of the Financial Services and Markets Act 2000 (‘FSMA’) sets out the general prohibition that no one may carry out a regulated activity unless they are either authorised or exempt?

A) Section 19 FSMA

B) Section 23 FSMA

C) Section 327 FSMA

A

A) Section 19 FSMA

(Correct. Section 19 FSMA contains the general prohibition. Section 23 relates to offences for contravening section 19. Section 327 relates to the exemption from the general prohibition)

153
Q

Which ONE of the following CORRECTLY summarises the conditions for carrying out an exempt regulated activity?

A) The person must be (i) a solicitor, (ii) must not receive payment or commission unless this is passed on to the client (or the client agrees otherwise), (iii) the activities must relate to a specified investment, and (iv) the person must comply with the SRA (Conduct of Business) Rules 2019.

B) The person must be (i) a member of a profession, (ii) must not receive payment or commission unless this is passed on to the client (or the client agrees otherwise), (iii) the activities must be specified in Part II RAO, and (iv) the person must comply with the FCA Rule Book.

C) The person must be (i) a member of a profession, (ii) must not receive payment or commission unless this is passed on to the client (or the client agrees otherwise), (iii) the activities must be incidental, and (iv) the person must comply with their relevant designated professional body’s rules.

A

C) The person must be (i) a member of a profession, (ii) must not receive payment or commission unless this is passed on to the client (or the client agrees otherwise), (iii) the activities must be incidental, and (iv) the person must comply with their relevant designated professional body’s rules.

(Correct. See section 327 Financial Services and Markets Act 2000 (FSMA))

154
Q

Which section of the Financial Services and Markets Act 2000 (‘FSMA’) defines a regulated activity?

A) Section 19

B) Section 22

C) Section 20

A

B) Section 22

(Correct. Section 22 FSMA defines a regulated activity as follows:
‘an activity is a regulated activity for the purposes of this Act if it is an activity of a specified kind which is carried on by way of business and – relates to an investment of a specified kind’)

155
Q

Which ONE of the following IS a specified investment under Part III of the Financial Services and Markets Act 2000 (Regulated Activities) Order 2001 (‘RAO’)?

A) Freehold property

B) A loan to a company to purchase an office

C) Rights under a contract of insurance

A

C) Rights under a contract of insurance

(Correct. Rights under a contract of insurance is a specified investment under article 75 RAO. A freehold property is not a financial product and is therefore not a specified investment under Part III RAO. A loan to a company to purchase an office does not fall within the definition of a regulated mortgage contract so is not a specified investment under Part III RAO)

156
Q

Which ONE of the following statements is CORRECT?

A) Advising a client to use the sale proceeds from their sale of shares in a company to purchase shares in British Gas plc would be a specified activity under article 53 RAO.

B) Arranging the sale of a client’s shares in a company to their business partner would be a specified activity under article 53 RAO.

C) Giving a client advice on the legal rights attaching to two different classes of shares in a company would be a specified activity under article 53 RAO.

A

A) Advising a client to use the sale proceeds from their sale of shares in a company to purchase shares in British Gas plc would be a specified activity under article 53 RAO.

(Correct. Advising a client to use the sale proceeds from their sale of shares in a company to purchase shares in British Gas plc would be a specified activity of advising on the merits of an investment under article 53 RAO because the advice would involve giving an opinion on the part of the solicitor and a recommendation as to a course of action.
Giving advice on the legal rights attaching to two different classes of shares in a company is generic advice, it does not require an element of opinion on the part of the solicitor and a recommendation as to a course of action, so it does not fall within advising on the merits of an investment under article 53.
Arranging the sale of a client’s shares in a company to their business partner would be a specified activity under article 25 (arranging deals in investments) but not under article 53 RAO)

157
Q

Which ONE of the following statements is CORRECT?

A) Step 3 of the FSMA decision tree is to work out whether the solicitor is providing an exempt regulated activity.

B) Step 3 of the FSMA decision tree is to work out whether the specified activity is one of the excluded activities under the RAO.

C) Step 3 of the FSMA decision tree is to work out whether the activity is specified under Part II RAO.

A

B) Step 3 of the FSMA decision tree is to work out whether the specified activity is one of the excluded activities under the RAO.

(Correct. Step 3 of the FSMA decision tree is to work out whether the specified activity is one of the excluded activities under the RAO.
Step 2 of the decision tree is to work out whether the activity is specified under Part II RAO.
Working out whether the solicitor is providing an exempt regulated activity forms part of step 4 of the FSMA decision tree)

158
Q

Which ONE of the following statements is CORRECT?

A) A solicitor who arranges for their client to buy 50% of the shares in a private company would NOT need to be authorised by the FCA because they fall within the exclusion in Article 70 RAO.

B) A solicitor who advises their client to buy 10% of the shares in ABC Limited would not need to be authorised by the FCA because they fall within the exclusion in Article 70 RAO.

C) A solicitor who sells 50% of their client’s shares in a company as agent for their client would need to be authorised by the FCA because they would not fall within the exclusion in Article 70.

A

A) A solicitor who arranges for their client to buy 50% of the shares in a private company would NOT need to be authorised by the FCA because they fall within the exclusion in Article 70 RAO

(Correct. The exclusion in Article 70 applies to the specified activities of arranging deals, advising and dealing as agent (which includes selling), so it could potentially apply to all three options. One of the conditions for Article 70 to apply is that the shares consist of or include 50 per cent or more of the voting shares in the company.
Arranging for your client to buy 50% of the shares meets this condition, so the exclusion WOULD apply and the solicitor would NOT need to be authorised by the FCA.
Advising your client to buy 10% of the shares does not meet this condition, so the exclusion would NOT apply and the solicitor WOULD need to be authorised by the FCA.
Selling 50% of your clients shares as agent meets this condition, so the exclusion WOULD apply and the solicitor would NOT need to be authorised by the FCA)

159
Q

Which ONE of the following statements is CORRECT?

A) A solicitor does not need to be authorised by the FCA to arrange for their client to buy shares in a company if the client is entering the transaction on advice given by an advisor who is authorised by the SRA.

B) A solicitor does not need to be authorised by the FCA to arrange for their client to buy shares in a company if the client is entering the transaction on advice given by a financial advisor who is authorised by the FCA.

C) A solicitor can only arrange for their client to buy shares in a company if the solicitor is authorised by the FCA or exempt.

A

B) A solicitor does not need to be authorised by the FCA to arrange for their client to buy shares in a company if the client is entering the transaction on advice given by a financial advisor who is authorised by the FCA

(Correct. Arranging for your client to buy shares in a company is a specified activity under Article 25(1) RAO. However if you read further, Article 29 RAO states that arrangements made by a person are excluded from Article 25(1) if they are entered on advice to the client by an authorised person. A financial advisor authorised by the FCA is an authorised person. Therefore under Article 25(1), the solicitor does not need to be authorised by the FCA to arrange for their client to buy the shares.
This conclusion is logical. If the client is acting on the advice of someone authorised by the FCA there is no additional need for the solicitor to be authorised by the FCA.
The financial advisor would need to be authorised by the FCA to advise under FSMA, not the SRA)

160
Q

Which section of the Financial Services and Markets Act 2000 (‘FSMA’) sets out the exemption from the general prohibition that no one may carry out a regulated activity unless they are either authorised or exempt?

A) Section 327 FSMA

B) Section 325 FSMA

C) Section 19 FSMA

A

A) Section 327 FSMA

(Correct. Section 327 sets out the conditions that must be satisfied for a person to meet the exemption from the general prohibition. The conditions include: the person must be (i) a member of a profession, (ii) must not receive payment or commission unless this is passed on to the client (or the client agrees otherwise), (iii) the activities must be incidental, and (iv) the person must comply with their relevant designated professional body’s rules. Section 19 FSMA contains the general prohibition. Section 325 relates to the FCA’s general duty)

161
Q

A property solicitor has recently completed the sale of a property for a client. The client asks the solicitor for advice on whether she should use the sale proceeds from the property to invest in shares in British Telecom PLC.

Which ONE of the following statements is CORRECT?

A) The solicitor cannot give the advice without breaching the general prohibition under s 19 FSMA because giving the advice would not be incidental for the purposes of s 327 FSMA.

B) Provided they comply with the rules set down by the SRA, the solicitor can give the advice without breaching the general prohibition under s 19 FSMA because the activity meets the conditions of s 327 FSMA and the SRA Financial Services (Scope) Rules 2019.

C) The solicitor cannot give the advice without breaching the general prohibition under s 19 FSMA because giving the advice would not arise out of or be complementary to the provision of a particular professional service to a particular client for the purposes of Rule 2 of the SRA Financial Services (Scope) Rules 2019.

A

C) The solicitor cannot give the advice without breaching the general prohibition under s 19 FSMA because giving the advice would not arise out of or be complementary to the provision of a particular professional service to a particular client for the purposes of Rule 2 of the SRA Financial Services (Scope) Rules 2019.

(Correct. Shares are a specified investment and the solicitor would be advising on the merits of investing in shares which is a specified activity. No exclusion applies. Giving the advice would fall within the condition of being incidental under s 327 FSMA because it would be a small part of the practice of the firm as a whole. However it would not be complementary for the purposes of Rule 2 of the SRA Financial Services (Scope) Rules 2019 because giving advice on whether to invest in shares in a particular company does not arise naturally out of the work the solicitor did in selling the property, it is a completely unrelated matter.
The solicitor should advise the client to take advice from a person authorised by the FCA, otherwise they would need to be authorised by the FCA to give the advice (which will require a lot of work on the part of the solicitor and their firm) or they will be in breach of s 19 FSMA which is a criminal offence)

162
Q

Which ONE of the following statements CORRECTLY sets out step 4 of the FSMA decision tree?

A) Does the activity fulfil the basic conditions in s 327 of FSMA and the SRA Financial Services (Conduct of Business) Rules 2019?

B) Does the activity fulfil the basic conditions in s 19 of FSMA and the SRA Financial Services (Conduct of Business) Rules 2019?

C) Does the activity fulfil the basic conditions in s 327 of FSMA and the SRA Financial Services (Scope) Rules 2019?

A

C) Does the activity fulfil the basic conditions in s 327 of FSMA and the SRA Financial Services (Scope) Rules 2019?

(Correct. This statement correctly summarises step 4 of the FSMA decision tree. If the activity does fulfil both these conditions, the solicitor will not need to be authorised by the FCA to carry out the regulated activity provided it complies with the rules set by the SRA)

163
Q

Which ONE of the following statements is CORRECT

A) It is a civil offence to offer shares to the public in the United Kingdom without issuing a prospectus approved by the FCA.

B) It is a criminal offence to offer shares to the public in the United Kingdom without issuing a prospectus approved by the relevant regulatory authority.

C) It is a criminal offence to offer shares to the public in the United Kingdom without issuing a prospectus approved by the SRA.

D) It is a civil offence to communicate a financial promotion unless you are authorised to do so by the SRA

A

B) It is a criminal offence to offer shares to the public in the United Kingdom without issuing a prospectus approved by the relevant regulatory authority

(Correct. Under s 85 FSMA it is a criminal offence to offer shares to the public without issuing a prospectus approved by the relevant regulatory authority. The relevant regulator in the UK is the FCA. In addition under s 21 FSMA it is a criminal offence to communicate a financial promotion unless you are authorised by the FCA or the financial promotion has been approved by a person authorised by the FCA)

164
Q

Which ONE of the following statements is CORRECT?

A) The exemption to the prohibition on communicating a financial promotion without the contents being approved by an authorised person that can be useful in practice is the necessary part exemption.

B) An exemption to the prohibition on communicating a financial promotion without the contents being approved by an authorised person that can be useful in practice is the sale of a body corporate exemption.

C) The prohibition on issuing financial promotions unless an authorised person has approved its content is rarely a problem in practice as it is a simple and cheap procedure to get the financial promotion approved by an authorised person

A

B) An exemption to the prohibition on communicating a financial promotion without the contents being approved by an authorised person that can be useful in practice is the sale of a body corporate exemption

(Correct. The sale of a body corporate exemption applies where the shares consist of 50% or more of the shares in the company or the buyer is buying day to day control of the company and certain conditions are met. This is can be a very useful exemption for solicitors acting on share sales. Necessary part is not an exemption to the prohibition on communicating a financial promotion without the contents being approved.
Getting a financial promotion approved by an authorised person can be time consuming and expensive for the client)

165
Q

Which ONE of the following statements is CORRECT?

A) An exemption to the rule that you cannot offer shares to the public without an approved prospectus arises where the offer is made to fewer than 150 persons.

B) An exemption to the rule that you cannot offer shares to the public without an approved prospectus arises where the offer is made to fewer than 100 persons who are qualified investors.

C) An exemption to the rule that you cannot offer shares to the public without an approved prospectus arises where the offer is made to fewer than 50 persons.

A

A) An exemption to the rule that you cannot offer shares to the public without an approved prospectus arises where the offer is made to fewer than 150 persons.

(Correct. If the offer is made to or directed at fewer than 150 persons, an approved prospectus is not required. The recipients do not need to be qualified investors)

166
Q

Whose responsibility does the SRA Risk Outlook say is it to keep client money safe?

A) The managing partner’s.

B) The compliance officer’s.

C) The SRA’s.

D) Every solicitor’s.

A

D) Every solicitor’s.

(Correct. The SRA Risk Outlook says that it every solicitor’s responsibility to keep client money safe)

167
Q

According to the SRA Risk Outlook, what can client money breaches lead to for solicitors?

A) They can lead to proceedings before the Solicitors Disciplinary Tribunal, with results that can include striking off.

B) The only sanctions client money breaches can lead to are fines.

C) There are no sanctions for solicitors for client money breaches, only for firms.

A

A) They can lead to proceedings before the Solicitors Disciplinary Tribunal, with results that can include striking off

(Correct. The SRA Risk Outlook states that: ‘We treat client money breaches particularly seriously. They can lead to proceedings before the SDT, with results that can include striking off.’ The SDT is the Solicitors Disciplinary Tribunal)

168
Q

Which one of the following is a recommendation made in respect of client money in the SRA Risk Outlook?

A) If client money is lost or taken, you must report this breach to the compliance officer where you work promptly, even where the money has been replaced already.

B) If client money is lost or taken, you must report this breach to the SRA promptly, even where the money has been replaced already.

C) If client money is lost or taken, you must report this breach to the Solicitors Disciplinary Tribunal within two working days, even where the money has been replaced already.

D) If client money is lost or taken, you report this breach to the SRA within seven working days, unless you have already replaced the money.

E) If client money is lost or taken, you must report this breach to the compliance officer where you work within two working days, unless you have already replaced the money.

A

B) If client money is lost or taken, you must report this breach to the SRA promptly, even where the money has been replaced already

(Correct. The SRA recommend that if client money is lost or taken, then you/the firm must report this breach to them promptly. This is the case even where you/the firm have already replaced the money)

169
Q

Which one of the following statements is correct with regard to accountants’ reports?

A) An accountant discovering a minor breach of the SRA Accounts Rules arising from an administrative error at a law firm will not necessarily have to prepare a qualified report in relation to that law firm’s accounts.

B) It is entirely up to a firm to decide the matters which are incorporated into the terms on which an accountant is engaged.

C) All law firms are obliged to submit their accountant’s reports to the SRA within six months of the end of that law firm’s accounting period.

A

A) An accountant discovering a minor breach of the SRA Accounts Rules arising from an administrative error at a law firm will not necessarily have to prepare a qualified report in relation to that law firm’s accounts

(Correct. SRA Guidance: ‘Planning for and completing an accountant’s report’ states that the SRA only expect reports to be qualified where there has been a significant breach of the Accounts Rules, such that client money is at risk. It also says that breaches arising from administrative errors are not likely to be significant but could be if they are persistent, derive from a failure of controls and have put client money at risk. The guidance states that the SRA recognise minor breaches of the Accounts Rules do occur in many firms and they are not expecting all identified breaches to be notified to them in the form of a qualified report)

170
Q

Which one of the following statements is correct with regard to accountants’ reports

A) If a firm’s balance of client money does not exceed an average of £10,000 and a maximum of £250,000 during an accounting period, it will not have to obtain an accountant’s report.

B) If a firm only holds client money which has been received during an accounting period from the Legal Aid Agency, it must obtain an accountant’s report.

C) No law firms are exempt from the requirement to obtain an accountant’s report.

A

A) If a firm’s balance of client money does not exceed an average of £10,000 and a maximum of £250,000 during an accounting period, it will not have to obtain an accountant’s report

(Correct. This is the exemption to Rule 12.1(a) set out in Rule 12.2(b))

171
Q

How long must an authorised body retain accounting records for?

A) Five years.

B) At least ten years from the end of the relevant accounting period.

C) At least six years.

D) One year after the end of the relevant accounting period.

A

C) At least six years

(Correct. This is the time period specified in Rule 13.1)

172
Q

Which of the following Part 2 Rules from the SRA’s Accounts Rules apply when a solicitor/authorised body is operating a bank account jointly with a client, when acting in a client’s matter?

A) All of the Rules in Part 2 of the SRA Accounts Rules.

B) Rule 8.2 (bank statements) and Rule 8.4 (bills of costs).

C) All of Rule 8 (on client accounting systems and controls) but not Rules 2-7.

D) Rule 8.2 (bank statements), Rule 8.3 (reconciliations) and Rule 8.4 (bills of costs).

A

B) Rule 8.2 (bank statements) and Rule 8.4 (bills of costs)

(Correct. Rule 9.1, on the operation of joint accounts, says that Part 2 of the SRA Accounts Rules does not apply to the joint account except for Rules 8.2 and 8.4)

173
Q

Which one of the following is correct with regard to operating a client’s own account as signatory?

A) Rule 10.1 on operation of a client’s own account applies whenever a solicitor/authorised body operates a client’s own account as signatory.

B) If, in the course of practice, a solicitor/authorised body operates a client’s own account as signatory, only Rule 8.2 (bank statements) and Rule 8.4 (bills of costs) from Part 2 of the SRA Accounts Rules apply.

C) An accountant’s report must be obtained for a client’s own account where the solicitor/authorised body is signatory unless one of the Rule 12.2 exceptions apply.

A

C) An accountant’s report must be obtained for a client’s own account where the solicitor/authorised body is signatory unless one of the Rule 12.2 exceptions apply.

(Correct. This is set out in Rule 12.1)

174
Q

What is a joint account in the context of Rule 9.1 of the SRA Accounts Rules?

A) A joint account in Rule 9.1 is an account which is in the name of two clients.

B) A joint account in Rule 9.1 is an account which is in the names of the solicitor/authorised body and the client or a third party.

C) A joint account in Rule 9.1 is an account which holds both client money and money belonging to the authorised body.

A

B) A joint account in Rule 9.1 is an account which is in the names of the solicitor/authorised body and the client or a third party

(Correct. See Rule 9.1)

175
Q

Which one of the following is a correct statement about Rule 11.1 of the SRA Accounts Rules regarding third party managed accounts?

A) Rule 11.1 provides that the fees for use of the third party managed account will be paid by the client.

B) Rule 11.1 only applies when the authorised body is delivering regulated services to the client.

C) Rule 11.1 allows an authorised body to receive and hold clients’ money in an account without Part 2 of the SRA Accounts Rules applying to the client money.

A

B) Rule 11.1 only applies when the authorised body is delivering regulated services to the client

(Correct. Rule 11.1 is clear that it covers use of a third party managed account for payments in respect of regulated services delivered by the authorised body to the client)

176
Q

Which one of the following is part of the definition of a third party managed account in the SRA Glossary?

A) The monies in the third party managed account must be owned beneficially by the client.

B) The third party is regulated by the SRA.

C) The third party managed account must be held at a bank or building society.

A

C) The third party managed account must be held at a bank or building society.

177
Q

Which one of the following statements about third party managed accounts is correct?

A) Money held in a third party managed account is not client money (as defined in the SRA Accounts Rules).

B) An authorised body does not have any obligations under the SRA Accounts Rules with regard to statements or records relating to any third party managed accounts.

C) An authorised body needs the SRA’s permission to use a third party managed account instead of/ as well as a client account.

A

A) Money held in a third party managed account is not client money (as defined in the SRA Accounts Rules)

(Correct. The SRA Guidance ‘Third party managed accounts’ explains that money held in a third party managed account does not fall under the definition of client money in the SRA Accounts Rules because it is not received or held by the authorised body. That the client’s money is not received or held by the authorised body is a requirement of Rule 11.1(a) on third party managed accounts)

178
Q

Which one of the following SRA Accounts Rules states that you must keep client money separate from money belonging to the authorised body?

A) Rule 4.2

B) Rule 3.2

C) Rule 3.1

D) Rule 4.1

A

D) Rule 4.1

(Correct. Rule 4.1 states that you must keep client money separate from money belonging to the authorised body)

179
Q

How quickly must client money be paid into a client account?

A) Within 7 days

B) As soon as practicable.

C) Within 2 working days

D) Promptly

A

D) Promptly

(Correct. Rule 3.2 states that client money must be paid promptly into a client account)

180
Q

Can client money be paid into the authorised body’s business account?

A) No, the SRA Accounts Rules do not permit this.

B) Yes, provided it is moved out of the business account and into the client account within 5 days.

C) Yes, if it is part of a mixed payment and is then promptly moved out of the business account and into the client account.

A

C) Yes, if it is part of a mixed payment and is then promptly moved out of the business account and into the client account.

(Correct. This is the position implied by Rule 4.2 which states that you must promptly allocate funds from mixed payments you receive to the correct client account or business account)

181
Q

Which one of the following statements, regarding the use of the client account, is correct under the Solicitors Accounts Rules?

A) Withdrawals from an authorised body’s client account must be in respect of the delivery of regulated services – your legal and professional services.

B) You may only withdraw money from the client account following receipt of instructions to do so from your client.

C) Provided you have acted on a transaction for a client, you may retain any remaining client money belonging to them in your client account, earning interest, until the client requests that the money is returned to them.

A

A) Withdrawals from an authorised body’s client account must be in respect of the delivery of regulated services – your legal and professional services.

(Correct. This is in Rule 3.3)

182
Q

In which one of the following scenarios can the solicitor properly withdraw money from their firm’s client account, in accordance with the Solicitors Accounts rules?

The client account currently holds £15,000, your client currently has £500 held in the client account. Their £500 was sent three days ago on account of costs generally for their property purchase transaction.

A) You withdraw £550 from the client account, on receipt of instructions from your client to pay Land Registry fees for registering their purchase of the property.

B) You withdraw £1,500 from the client account to pay stamp duty land tax on your client’s property purchase.

C) You withdraw £250 from the client account to pay for the desktop environmental search which needs to be carried out in connection with the client’s property purchase.

A

C) You withdraw £250 from the client account to pay for the desktop environmental search which needs to be carried out in connection with the client’s property purchase.

(Correct. Under Rule 5.1 this would fall within reason (a) for being able to withdraw money from the client account – that you are withdrawing the money for the purpose for which it is being held (generally on account of costs involved in their transaction). The client also has sufficient funds for this payment in the client account (Rule 5.3))

183
Q

Which one of the following would be in accordance with the Solicitors Accounts Rules?

A) Transferring money from the client account to your business account to pay a bill which has been sent to the client where they have authorised the transfer of the costs and have sufficient funds in the client account.

B) Paying a mixed payment into your business account and leaving the client money element in the business account for seven days whilst you prepare and send a bill to the client, to prevent the need to transfer the client money element into the client account and then back to the business account to pay the bill later.

C) Transferring money from the client account to your business account following a phone call with your client where you let them know that you would shortly be sending them a bill and they authorised the transfer of costs and have sufficient funds in the client account.

A

A) Transferring money from the client account to your business account to pay a bill which has been sent to the client where they have authorised the transfer of the costs and have sufficient funds in the client account.

(Correct. Transferring money from the client account to your business account to pay a bill which has been sent to the client where they have authorised the transfer of the costs and have sufficient funds in the client account is in accordance with the SRA Accounts Rules)

184
Q

Which one of the following statements is incorrect?

A) The usual way in which legislation comes into operation is through a commencement provision in the relevant Act, giving the relevant Secretary of State the power to name a commencement date by issuing a statutory instrument (‘S.I.’)

B) A Private Act of Parliament means that it has been drafted by an individual MP on either side of the House without official government sponsorship.

C) The front ‘page’ of an Act of Parliament contains both a short and a long (explanatory) title as well as the date of the royal assent.

D) If passed by Parliament, government bills and private members’ bills ultimately enjoy the same legal status. However, the prospects of the former being enacted are far higher than the latter, as they represent official government policy and (as long as the ruling party has a majority) have the weight of the government party’s voting block behind them.

E) Acts of Parliament are divided up into different ‘parts’, each of which will have a larger number of ‘sections’ and often ‘sub-sections’.

A

B) A Private Act of Parliament means that it has been drafted by an individual MP on either side of the House without official government sponsorship

(Correct. This confuses a Private Act with a successful private member’s bill. A Private Act should instead be contrasted with a Public Act, the latter relating to a matter of general public policy and the former to a specific, often local need for statutory approval, such as a large infrastructure project)

185
Q

Which one of the following statements in relation to Royal Assent is correct?

A) Royal Assent is governed by the Royal Assent Act 1967. On the date of Royal Assent, a bill becomes an Act of Parliament and may not come into force until later.

B) Royal Assent is a formality and a bill does not become an Act of Parliament until in comes into force. This is not necessarily at the date of Royal Assent.

C) Royal Assent is a formality and on receiving Royal Assent a bill becomes an Act of Parliament and immediately comes into force.

D) Royal Assent is a formality governed by the Royal Assent Act 1967. A monarch can refuse to grant Royal Assent and refer a bill back to Parliament.

E) Royal Assent is governed by the Royal Assent Act 1967. On the date of Royal Assent, a bill becomes an Act of Parliament and immediately comes into force.

A

A) Royal Assent is governed by the Royal Assent Act 1967. On the date of Royal Assent, a bill becomes an Act of Parliament and may not come into force until later.

186
Q

Which one of the following statements is incorrect?

A) A bill may only be introduced into the House of Commons.

B) The second reading provides an opportunity for the relevant part of Parliament to discuss the amendments made to the bill by the other chamber.

C) The Committee stage of a bill takes place after the second reading.

D) The House of Lords’ powers are ultimately restricted to the review, proposed amendment and delay of legislation; an outright veto is not possible.

E) The first reading is a mere formality where the bill’s title is announced.

A

A) A bill may only be introduced into the House of Commons

(Correct. A bill can be introduced in either House, though certain parliamentary rules or conventions determine which House in certain situations, for example bills on taxation)

187
Q

Which one of the following statements is incorrect?

A) In addition to so-called ‘rules’ of statutory interpretation, the courts can apply certain ‘linguistic presumptions’ to aid them in the proper construction of legislation.

B) The modern judiciary approaches the task of statutory interpretation with an open mind both to the ordinary meaning of words in legislation and to the wider and more contextual purpose of the Act in question.

C) It is fair to say that some of the older terminology, notably the so-called ‘mischief rule’, has effectively become redundant in the modern age, and is only of historic significance.

D) The so-called ‘rules’ of statutory interpretation are essentially matters for academic debate and are not addressed or referred to as such in actual court judgments.

E) Constitutional responsibility for interpreting statute rests jointly with Parliament and the judiciary.

A

E) Constitutional responsibility for interpreting statute rests jointly with Parliament and the judiciary.

(Correct. This is not correct – once legislation has been passed by Parliament, it is the constitutional function of the courts to determine its meaning)

188
Q

Which one of the following statements is incorrect?

A) The four established rules of statutory interpretation have more or less equal relevance and applicability in modern cases.

B) The old ‘mischief rule’, with its emphasis on the prior problem in the law that the relevant legislation was designed to solve, has to all intents and purposes been subsumed within the modern purposive approach.

C) The modern purposive approach has partially been influenced by the culture of European jurisprudence and its generally teleological approach.

D) In the case of Adler v George, it was the absurdity of a literal reading of the legislative provision that engaged the “golden rule” and persuaded the court to imply a more logical and sensible reading of s. 3 of the Official Secrets Act 1920.

E) The purposive approach to statutory interpretation is employed across the board nowadays and is equally applicable to all areas of societal regulation.

A

E) The purposive approach to statutory interpretation is employed across the board nowadays and is equally applicable to all areas of societal regulation.

(Correct. Though it is clear that this contextual approach is far more common in the modern era, the scope of its application will be varied by the courts, depending on the nature of the legal question before it)

189
Q

Which one of the following statements is correct?

A) Where general words follow specific words, the presumption of ejusdem generis means that the general words are interpreted so as to restrict them to the same kind of matters or objects as the preceding specific words

B) The Latin phrase ejusdem generis refers to the situation in which the court will consider words in a particular provision in the context of the Act itself.

C) The case of Inland Revenue Commissioners v Frere the court was able to interpret what other forms of payment were referred to in s. 169 of the relevant Act in relation to the phrase: “interest, annuities and other annual payments”.

D) The use by the courts of linguistic presumptions is very much a feature of cases approached in a purposive way

E) The Latin phrase expressio unius est exclusio alterius relates to the court considering open lists containing words of a similar kind or nature

A

A) Where general words follow specific words, the presumption of ejusdem generis means that the general words are interpreted so as to restrict them to the same kind of matters or objects as the preceding specific words

(Correct. Check your materials on statutory interpretation)

190
Q

You have acted for a client whose transaction has now finished. You are not currently acting on any other matters for them although they are a long-standing client. There is some client money belonging to them in the client account. Which one of the following best sets out what you should do with this?

A) You must send the money to the client and the relevant accounting entries are as follows:
Credit client ledger
Debit cash sheet client account.

B) You must send the money to the client and the relevant accounting entries are as follows:
Debit client ledger
Credit cash sheet client account.

C) You must send the money to the client and the relevant accounting entries are as follows:
Credit client ledger business side
Debit cash sheet business account.

D) There is no need to send the money to the client. It can remain in your client account until you are next instructed by the client.

A

B) You must send the money to the client and the relevant accounting entries are as follows:
Debit client ledger
Credit cash sheet client account

(Correct. Rule 2.5 states that you must ensure that client money is returned promptly to the client for whom it’s held, as soon as there is no longer any proper reason to hold those funds. These are the correct accounting entries for withdrawing money from the client account to send it (in a cheque or by electronic transfer) to the client)

191
Q

A client has no money in the client account. Their solicitor pays a Land Registry fee on their behalf. Which one of the following correctly sets out the accounting entries to record this?

A) Debit client ledger client account
Credit cash sheet client account.

B) Credit client ledger client account
Debit cash sheet client account.

C) Credit client ledger business account
Debit cash sheet business account.

D) Debit client ledger business account
Credit cash sheet business account.

A

D) Debit client ledger business account
Credit cash sheet business account.

(Correct. These are the correct entries to record the payment out of non-client money. Non-client money will be used to make the payment because the client does not have any client money in the client account and Rule 5.3 states that you only withdraw client money from a client account if sufficient funds are held on behalf of that client to make the payment)

192
Q

You act for a client who has no money in the client account. You use cash from petty cash to pay a taxi fare to attend a meeting with your client. Your client later sends you a cheque to reimburse you for the taxi fare. Which one of the following correctly sets out the accounting entries to record the receipt of the cheque?

A) Debit client ledger business account
Credit cash sheet petty cash account

B) Credit client ledger client account
Debit cash sheet client account.

C) Credit client ledger business account
Debit cash sheet business account.

A

C) Credit client ledger business account
Debit cash sheet business account.

(Correct. When the cheque reimbursing you is received, the entries needed are these ones to record the receipt of non-client money into the business account (because the cheque cannot be paid into petty cash as it is not an actual bank account))

193
Q

Your client has £1500 in the client account. You have just completed work on their transaction which is now complete. Your costs (fees for your work on the matter) come to £995. You have drawn up a bill of costs ready to send to the client. Which one of the following best sets out your position?

A) You cannot transfer £995 from the client account to the business account to pay your costs until the bill of costs has been given to the client.

B) You can transfer £995 from the client account to the business account using the following entries:
Debit client ledger client account
Credit cash sheet client account
Credit client ledger business account
Debit cash sheet business account

C) You can transfer £995 from the client account to the business account using the following entries:
Debit client ledger client account
Credit cash sheet client account.

D) You can transfer £995 from the client account to the business account using the following entries:
Debit client ledger client account
Credit client ledger business account

A

A) You cannot transfer £995 from the client account to the business account to pay your costs until the bill of costs has been given to the client

(Correct. See Rule 4.3(a) and (b))

194
Q

You have just discovered that a payment was made from the client account on behalf of a client who did not actually have any money in the client account. Which one of the following best sets out what you should do?

A) You should within five working days replace the client money withdrawn by doing the following:
Debit client ledger client account
Credit cash sheet client account
Credit client ledger business account
Debit cash sheet business account.

B) You should promptly replace the client money withdrawn by doing the following:
Debit client ledger client account
Credit cash sheet client account
Credit client ledger business account
Debit cash sheet business account.

C) You should promptly replace the client money withdrawn by doing the following:
Debit client ledger business account
Credit cash sheet business account
Credit client ledger client account
Debit cash sheet client account.

D) You must report this breach to your authorised body’s accounts officer within five working days.

A

C) You should promptly replace the client money withdrawn by doing the following:
Debit client ledger business account
Credit cash sheet business account
Credit client ledger client account
Debit cash sheet client account.

(Correct. Under Rule 6.1, any breaches of the SRA Accounts Rules must be corrected promptly upon discovery. Money improperly withdrawn from a client account must be immediately replaced. These are the correct entries to transfer money from the business account and pay it into the client account to replace the money improperly withdrawn)

195
Q

You act for a bank which is providing a mortgage to another of your clients who is buying a property. The bank has the amount of the mortgage money within the client account and all necessary authorities and authorisations have been obtained to now advance the mortgage money to the buyer.

Which one of the following sets of entries is the correct set to record this transfer of money?

A) If a law firm receives a cheque from a client which includes both client money and non-client money, the full amount must be paid in its entirety into the firm’s client account and the non-client money must then be promptly transferred to the firm’s business account.

B) If a law firm receives a cheque from a client which includes both client money and non-client money, the full amount must be paid in its entirety into the firm’s business account and the client money must then be promptly transferred to the firm’s client account.

C) If a law firm receives a cheque from a client which includes both client money and non-client money, it is possible for the law firm to instruct their bank to ‘split’ the cheque and pay part of the money into the firm’s client account and part of the money into their business account.

A

C) If a law firm receives a cheque from a client which includes both client money and non-client money, it is possible for the law firm to instruct their bank to ‘split’ the cheque and pay part of the money into the firm’s client account and part of the money into their business account.

(Correct. Such a cheque would constitute a mixed payment. The accounting entries involved are:
Credit the client ledger client account of the client; and
Debit the cash sheet client account with the client money element of the payment and
Credit the client ledger business account of the client and debit the cash sheet business account with the non-client money element of the payment)

196
Q

Which ONE of the following statements correctly completes the sentence below:

If you act for two clients and transfer money from one client to another, the following entries are made:

A) Debit the client ledger client account for the client who is transferring the money and credit the cash sheet business account; and
Credit the client ledger client account for the client to whom the money is being transferred, and debit the cash sheet business account

B) Debit the client ledger client account for the client who is transferring the money and credit the cash sheet client account.

C) Debit the client ledger client account of the client who is transferring the money and credit the client ledger client account of the client to whom the money is being transferred.

D) Debit the client ledger client account for the client who is transferring the money and credit the cash sheet client account; and
Credit the client ledger client account for the client to whom the money is being transferred, and debit the cash sheet client account

A

C) Debit the client ledger client account of the client who is transferring the money and credit the client ledger client account of the client to whom the money is being transferred

(Correct. If you act for two clients and a transfer of client money is being made from one client to the other, the client money being transferred is already in the firm’s client account. There is no need to take the money out of the client account and then pay it back into the same account. You can transfer money between two clients of the firm in one step)

197
Q

You act for the executors on the administration of an estate and also for the sole beneficiary. The executors have the funds from winding up the estate within the client account, have made all necessary payments out of the estate and all necessary authorities and authorisations have been obtained to now pay the legacy to the beneficiary.

Which one of the following sets of entries is the correct set to record the transfer of money from the executors to the beneficiary?

A) Debit client ledger client account of the executors
Credit client ledger client account of the beneficiary.

B) Debit the client ledger client account for the executors and credit the cash sheet business account; and
Credit the client ledger client account for the beneficiary, and debit the cash sheet business account

C) Debit the client ledger client account for the executors and credit the cash sheet client account; and
Credit the client ledger client account for the beneficiary, and debit the cash sheet client account

A

A) Debit client ledger client account of the executors
Credit client ledger client account of the beneficiary

(Correct. As both the executors and the beneficiary are your clients and the money is already in the client account, only one step is required to record this and show that the money in the client account is now held for the beneficiary rather than the executors. There will not be an entry in the client account as there is no need to withdraw the money from the client account and then pay it back in)

198
Q

You act for a bank which is providing a mortgage to another of your clients who is buying a property. The bank has the amount of the mortgage money within the client account and all necessary authorities and authorisations have been obtained to now advance the mortgage money to the buyer.

Which one of the following sets of entries is the correct set to record this transfer of money?

A) Debit client ledger client account of the bank
Credit client ledger client account of the buyer.

B) Debit client ledger client account of the bank
Credit cash sheet client account
Credit client ledger business account of the buyer
Debit cash sheet business account.

C) Debit client ledger client account of the buyer
Credit client ledger client account of the bank.

A

A) Debit client ledger client account of the bank
Credit client ledger client account of the buyer

(Correct. As both the bank and the buyer are your clients and the money is already in the client account, only one step is required to record this and show that the money in the client account is now held for the buyer rather than the bank. There will not be an entry in the client account as there is no need to withdraw the money from the client account and then pay it back in)

199
Q

A firm of solicitors is acting on behalf of a client who is buying a property. A deposit of £25,000 has been received from your client, in respect of the purchase, which will be held as stakeholder.

Which one of the following pairs of double entries shows how the receipt of the deposit should be recorded?

A) Credit stakeholder ledger
Debit cash sheet client account

B) Credit client ledger client account
Debit cash sheet client account

C) Debit client ledger client account
Credit cash sheet client account

A

B) Credit client ledger client account
Debit cash sheet client account

(Correct. The accounting entries are simply the usual ones for client money received. The deposit is client money under Rule 2.1(a) and the fact the deposit will be held as stakeholder is only relevant to the seller’s solicitor. In this question we are the buyer’s solicitor and how the deposit will be held by the seller’s solicitor once it has been paid to them is irrelevant to our accounting entries)

200
Q

A firm of solicitors is acting on behalf of a client who is selling his property. A deposit of £25,000 has been received in respect of the sale and is held as stakeholder.

Which one of the following pairs of double entries shows how the receipt of the deposit should be recorded?

A) Credit client ledger client account
Debit stakeholder ledger

B) Credit cash sheet client account
Debit stakeholder ledger

C) Credit cash sheet business account
Debit stakeholder ledger

D) Credit cash sheet client account
Debit client ledger client account

E) Credit stakeholder ledger
Debit cash sheet client account

A

E) Credit stakeholder ledger
Debit cash sheet client account

(Correct. These are the accounting entries to record the receipt by the seller’s solicitor of a deposit, to be held as stakeholder)

201
Q

A firm of solicitors is acting on behalf of a client who is buying a property. The firm also acts for the lender who will be providing a mortgage to the buyer to fund the purchase. The firm does not have a separate client ledger for the lender.

Which one of the following pairs of double entries should the firm make to record the receipt of funds from the lender to purchase the property?

A) Credit client ledger client account for the (buyer) client
Debit cash sheet client account

B) Credit client ledger client account for the (lender) client
Debit cash sheet client account
Debit client ledger client account for the (lender) client
Credit client ledger client account for the (buyer) client

C) Credit client ledger client account for the (lender) client
Debit cash sheet client account

D) Debit client ledger client account for the (buyer) client
Credit cash sheet client account

E) Debit client ledger client account for the (lender) client
Credit client ledger client account for the (buyer) client

A

A) Credit client ledger client account for the (buyer) client
Debit cash sheet client account

(Correct. In this case the firm does not have a separate client ledger for the lender so the entries will be the normal entries for the receipt of client money into the client account for the buyer. The details in the ledger would state that the mortgage funds have come from the lender)

202
Q

A firm of solicitors is acting on behalf of a client who is selling his property. An outstanding mortgage on the property is paid off at completion of the sale using the completion monies received in the client account. The firm of solicitors acts for the seller’s lender and has a separate client ledger for the lender.

Which one of the following pairs of double entries will be included in those which record the redemption of the seller’s mortgage?

A) Debit client ledger client account seller
Credit client ledger client account lender

B) Debit client ledger client account lender
Credit cash sheet business account

C) Credit client ledger client account lender
Credit cash sheet client account

A

A) Debit client ledger client account seller
Credit client ledger client account lender

(Correct. These are the entries showing the transfer of the redemption money (which is client money and already in the client account) between clients (from the seller to the lender). After this step, the redemption money will be paid out to the lender using the client money paid out entries)

203
Q

A firm of solicitors holds £50,000 in a separate designated client account for their client. The separate designated client account is closed and the £50,000 transferred back into the main client account.

Which one of the following shows the correct entries used to record the transfer?

A) Debit client ledger client account for the client
Credit cash sheet client account
Credit client ledger separate designated client account for the client
Debit cash sheet separate designated client account for the client

B) Credit client ledger separate designated client account for the client
Debit client ledger client account for the client

C) Debit client ledger separate designated client account for the client
Credit cash sheet separate designated client account for the client
Credit client ledger client account for the client
Debit cash sheet client account

D) Debit client ledger separate designated client account for the client
Credit client ledger client account for the client

A

C) Debit client ledger separate designated client account for the client
Credit cash sheet separate designated client account for the client
Credit client ledger client account for the client
Debit cash sheet client account

(Correct. The transfer is from one bank account (the separate designated account) to another (the general/main client account) and so it involves two double entries ie four entries in total. One pair of entries to record taking the money out of the separate designated client account and the second pair of entries to record the paying in of the money to the general/main client account)

204
Q

Can a firm of solicitors agree with a client that it will never pay interest on their client money whilst it is in the firm’s client account?

Which one of the following best answers this question?

A) Yes, provided that the client agrees and sufficient information has been provided to the client to enable them to give their informed consent.

B) Yes, provided that a written agreement is entered into with the client as to interest not being paid and provided that sufficient information has been provided to the client to enable them to give informed consent.

C) No, this is prohibited by the SRA Accounts Rules.

A

B) Yes, provided that a written agreement is entered into with the client as to interest not being paid and provided that sufficient information has been provided to the client to enable them to give informed consent

(Correct. Rule 7.1 provides that a fair sum of interest on client money held by the firm must be paid to the client. Rule 7.2 allows the firm and the client to come to a different arrangement as to the payment of interest but this must be by written agreement and the firm must provide sufficient information to enable the client to give informed consent)

205
Q

A firm of solicitors is acting on behalf of a client in a litigation matter. The claim by the client has now been settled and damages have been received by the firm. The firm retains the funds on client account for two weeks whilst the client has a holiday. The firm pays the client interest for the two weeks it has held the funds.

Which one of the following should be included in the entries that record the payment of the interest?

A) Credit interest payable ledger
Credit cash sheet client account

B) Credit cash sheet client account
Debit cash sheet business account

C) Debit interest payable ledger
Credit cash sheet business account

D) Credit interest payable ledger
Debit cash sheet business account

A

C) Debit interest payable ledger
Credit cash sheet business account

(Correct. These entries record the withdrawing of the interest monies from the firm’s business account which is the first step in the transfer of the interest monies from the firm’s business account to their client account and happens prior to the second step which is the payment of the interest monies into the client account)

206
Q

Which one of the following statements regarding the SRA Accounts rules on bills and accounting entries for issuing a bill plus VAT is correct?

A) There is no obligation for an authorised body to keep a record of all bills or other written notifications of costs but it is advisable.

B) One of the accounting entries for issuing a bill plus VAT will be recorded in the cash sheet client account.

C) One of the accounting entries for issuing a bill plus VAT will be recorded in the client ledger client account of the client being sent the bill.

D) The VAT account [ledger] entry to record the VAT charged on the profit costs in the bill will be a credit.

A

D) The VAT account [ledger] entry to record the VAT charged on the profit costs in the bill will be a credit.

207
Q

A bill was sent to your client for profit costs of £1,000 plus VAT of £200. The client has now sent you a cheque for £1,200 in payment of the bill plus VAT. Which one of the following pairs of double entries shows how the payment should be recorded?

A) Debit client ledger business account
Credit cash sheet business account

B) Debit client ledger client account
Credit profit costs account

C) Credit client ledger business account
Debit cash sheet business account

D) Credit client ledger business account
Debit profit costs account

A

C) Credit client ledger business account
Debit cash sheet business account

(Correct. These are the accounting entries for recording the receipt of non-client money (which the cheque in payment of the bill is))

208
Q

A law firm is due to issue a bill to a lender client for £1000 plus £200 VAT for the firm’s costs associated with the purchase of a property. The firm has a separate client ledger for the lender.

Which one of the following options shows the entries the firm should make when it issues the bill?

A) Debit client ledger business account for the borrower £1000
Credit profit costs £1000
Debit client ledger business account for the borrower £200
Credit VAT account £200

B) Debit client ledger business account for the lender £1000
Credit profit costs £1000
Debit client ledger business account for the lender £200
Credit VAT account £200

C) Debit client ledger business account for the borrower £1200
Credit profit costs £1200

D) Credit client ledger business account for the borrower £1200
Debit cash sheet business account £1200

A

B) Debit client ledger business account for the lender £1000
Credit profit costs £1000
Debit client ledger business account for the lender £200
Credit VAT account £200

(Correct. Because the firm has a separate client ledger for the ledger, the entries will be made in the client ledger business account for the lender, plus the profit costs and VAT accounts)

209
Q

Which one of the following statements about the accounting entries to record the issuing and/or paying of a bill is correct?

A) If a borrower client has agreed to pay their lender’s costs (you also act for the lender and have opened a separate client ledger for them), the accounting entries involved in recording the transfer of the costs will include an entry being made in the cash sheet client account.

B) If a bill is issued to a borrower client’s lender (for whom you also act) and you do not have a separate client ledger for the lender, the accounting entries for the issue of the bill will include records being made in the borrower client’s client ledger client account.

C) A bill is issued to a borrower client’s lender (for whom you also act) and you do not have a separate client ledger for the lender. The borrower client agrees to pay the lender’s costs by a transfer from the client account. There will be four (two pairs) accounting entries to record the transfer.

A

C) A bill is issued to a borrower client’s lender (for whom you also act) and you do not have a separate client ledger for the lender. The borrower client agrees to pay the lender’s costs by a transfer from the client account. There will be four (two pairs) accounting entries to record the transfer.

(Correct. The issue of the bill plus VAT will be recorded in the borrower client’s client ledger and the transfer from the client account to the business account to pay the debt will involve four entries (as it is a transfer from one bank account to another). The entries would be:
Debit client ledger client account
Credit cash sheet client account
Credit client ledger business account
Debit cash sheet business account
In each entry the amount transferred will be the total of the bill plus VAT (as one figure))

210
Q

Your firm acts for the lender and the borrower on the purchase of a property. Your firm sent a bill to the lender for profit costs of £5,000 plus VAT of £1000. The firm does not have a separate client ledger for the lender. The borrower client has £10,000 in their client account with the firm and has instructed you to pay the lender’s bill by transferring the money from their client account.

Which one of the following options shows the entries to record the transfer?

A) Debit client ledger client account for the borrower £6000
Credit cash sheet client account £6000
Credit client ledger business account for the borrower £6000
Debit cash sheet business account £6000

B) Credit client ledger business account for the borrower £6000
Debit cash sheet business account £6000

C) Credit client ledger business account for the borrower £5000
Debit profit costs £5000
Credit client ledger business account for the borrower £1000
Debit VAT account £1000

D) Debit client ledger client account for the lender £6000
Credit cash sheet client account £6000
Credit client ledger business account for the lender £6000
debit cash sheet business account £6000

E) Credit client ledger business account for the borrower £6000
Debit profit costs account £6000

A

A) Debit client ledger client account for the borrower £6000
Credit cash sheet client account £6000
Credit client ledger business account for the borrower £6000
Debit cash sheet business account £6000

(Correct. As the firm does not have a separate client ledger for the lender, the issue of the bill plus VAT will have been recorded in the borrower client’s client ledger business account, because the profit costs is money due to the firm, not client money. Paying the bill by a transfer from the borrower’s client account involves two steps: (1) money needs to be transferred out of the borrower’s client account (debit client ledger client account, and credit cash sheet client account); and (2) the money needs to be paid into the firm’s business account (credit client ledger business account, and debit cash sheet business account).
In each entry the amount transferred will be the total of the bill plus VAT (as one figure))

211
Q

You are a solicitor in a firm which acts for a client on tax matters. You sent a bill for fees of £1800 plus VAT to the client a week ago and after discussing the bill with the client the managing partner of your firm has agreed to reduce the fees by £300 plus VAT.

Which of the following pairs of double entries show how the abatement should be recorded?

A) Debit client ledger business account £360
Credit profit costs £360

B) Debit client ledger business account £300
Credit profit costs £300
Debit client ledger business account £60
Credit VAT account £60

C) Credit client ledger business account £300
Debit profit costs £300
Credit client ledger business account £60
Debit VAT account £60

D) Credit client ledger business account £360
Debit profit costs £360

A

C) Credit client ledger business account £300
Debit profit costs £300
Credit client ledger business account £60
Debit VAT account £60

212
Q

You are a solicitor in a firm which acts for a retail client. A year ago you sent a bill of costs to the client for property work on their lease. The bill was for £2800 plus VAT. You have just learnt that the client has gone into liquidation and creditors are unlikely to recover their unpaid debts. Your firm decides to write off the debt as no client money is held for the client.

Which of the following pairs of double entries show how the writing off of the debt should be recorded?

A) Credit client ledger business account £3360
Debit bad and doubtful debts £2800
Debit VAT Account £560

B) Debit client ledger client account £3360
Credit bad and doubtful debts £2800
Credit VAT Account £560

C) Debit client ledger business account £2800
Credit bad and doubtful debts £2800
Debit client ledger business account £560
Debit VAT account £560

D) Credit client ledger business account £2800
Debit bad and doubtful debts £2800
Credit client ledger business account £560
Debit VAT Account £560

A

D) Credit client ledger business account £2800
Debit bad and doubtful debts £2800
Credit client ledger business account £560
Debit VAT Account £560

(Correct. These are the correct entries for the correct figures. The VAT on the bill of £2800 is £560 (20%). The debt and VAT need to be written off separately in separate double entries)

213
Q

Which one of the following best sets out the accounts/ledgers where records will be made to record the reduction of a bill for fees of £600 plus VAT that had previously been sent to a client?

A) Profit costs and VAT account.

B) Client ledger client account, profit costs and VAT account.

C) Client ledger client account and profit costs.

D) Client ledger business account, profit costs and VAT account.

E) Client ledger client account, client ledger business account and profit costs.

A

D) Client ledger business account, profit costs and VAT account

(Correct. Two entries are made in the client ledger business account and one each in profit costs and VAT account to record the reduction of the profit costs and VAT)

214
Q

Which one of the following statements about paying a disbursement plus VAT as principal/agent is correct?

A) When paying a disbursement plus VAT as agent, there are no separate VAT entries; the payment is for the total of the cost plus the VAT.

B) When paying a disbursement plus VAT as principal, there are no separate VAT entries; the payment is for the total of the cost plus the VAT.

C) When paying a disbursement plus VAT as agent, the payment must be made out of the business account.

D) When paying a disbursement plus VAT as principal, the payment must be made out of the client account.

A

A) When paying a disbursement plus VAT as agent, there are no separate VAT entries; the payment is for the total of the cost plus the VAT.

215
Q

A solicitor instructed an environmental search agency to conduct an environmental search on a property for a client. The client at that time had £1000 in the client account generally on account of costs. The search agency’s invoice for £600 plus VAT arrived, addressed to the solicitor. The solicitor paid the invoice.

Which one of the following statements about this scenario would be correct?

A) The solicitor recorded an entry for £120 in the VAT account [ledger] when making the payment.

B) The solicitor recorded an entry for £720 in the client ledger business account when making the payment.

C) The solicitor paid the search agency’s invoice as agent of the client.

D) The solicitor paid £720 from the client account to pay the invoice.

A

A) The solicitor recorded an entry for £120 in the VAT account [ledger] when making the payment

(Correct. The solicitor was paying the invoice using the principal method as the solicitor was acting as principal (the solicitor instructed the agency and the invoice was addressed to the solicitor). When paying as principal there are separate accounting entries for the cost and for the VAT. The VAT accounting entries involve recording the VAT in the VAT account [ledger] and in the cash sheet business account)

216
Q

A solicitor instructs an accountant to prepare a report which the solicitor will use to advise their client. The accountant sends their invoice for £800 plus VAT addressed to the solicitor and the solicitor pays it.

Which one of the following options correctly sets out the accounting entries that record payment of the invoice?

A) Credit VAT account £160
Debit cash sheet business account £160
Credit client ledger business account £800
Debit client ledger business account £800

B) Credit client ledger business account £960
Debit cash sheet business account £960

C) Debit client ledger business account £800
Credit cash sheet business account £800
Debit VAT account £160
Credit cash sheet business account £160

D) Debit client ledger client account £960
Credit cash sheet client account £960

E) Debit client ledger client account £800
Credit cash sheet client account £800
Debit VAT account £160
Credit cash sheet client account £160

A

C) Debit client ledger business account £800
Credit cash sheet business account £800
Debit VAT account £160
Credit cash sheet business account £160

(Correct. The solicitor is paying the invoice as principal. The VAT and the cost are kept separate and the payment is made from the business account)

217
Q

You are acting for a client on the purchase of a property. Your client has instructed a surveyor who sends an invoice for £2000 plus VAT to your firm. The invoice is addressed to your client. Your client has £5000 in your firm’s client account on account of costs generally. The client asks you to pay the invoice on her behalf.

Which one of the following options correctly sets out the accounting entries that record payment of the invoice?

A) Debit client ledger client account £2000
Credit cash sheet client account £2000

B) Debit client ledger client account £2400
Credit cash sheet client account £2400

C) Debit client ledger business account £2400
Credit cash sheet business account £2400

D) Debit client ledger client account £2000
Credit cash sheet business account £2000
Debit VAT account £400
Credit cash sheet business account £400

E) Debit client ledger business account £2000
Credit cash sheet business account £2000
Debit VAT account £400
Credit cash sheet business account £400

A

B) Debit client ledger client account £2400
Credit cash sheet client account £2400

(Correct. The solicitor will be paying the invoice as agent because the client instructed the surveyor and the invoice was addressed to the client. The cost and the VAT are added together and there is no separate entry for the VAT element. The firm holds sufficient money on client account for the client (£5000) and therefore payment should be made out of the law firm’s client account. The payment will be recorded as a credit in the cash sheet client account and a debit in the client ledger client account (showing that the solicitor now holds less money for the client))